Sunteți pe pagina 1din 41

XVIII.

Neurologic disorders IOS: Done PIR: To do PREP: Done somewhere

A. Signs and symptoms of neurologic dysfunction


1. Headache: Ddx: Viral Infxn, Migraine, Neoplasm
• Recognize the physical characteristics of a headache due to increase intracranial pressure-
2007:22
• Know the elements of history that characterize a migraine 2007;28:43
• Be aware of neurologic defects that can be associated with a migraine 2007:22

• Know the elements of history that characterize a headache due to stress/tension/emotion- 2009:51
• Know that a headache can be caused by depression 2009:51

Red flags for inc ICP: worsening pain at night or immediately upon awakening, vomiting, worsening pain with
coughing or straining, Papilledema, focal neurologic findings

Migraine headaches are periodic, may have an aura, are relieved by sleep, FH often +
Complicated migraines can be accompanied by focal neurologic deficits such as hemiparesis, cranial nerve palsies,
and visual/oculomotor or sensory disturbances, or dysphasia.
Migraine without aura: frontal or temporal intense headache, lasts from 1-48 hr, assoc. w. nausea, vomiting,
photophobia, phonophobia; cause avoidance of routine phys activity. .60-85% all migraines are w/o aura.

Pain from stress-related or tension headache generally is diffuse and may be described as "bandlike" or throbbing.
Pain usually occurs on most days, and school absence is frequent.

Family stressors and depression are causes of headache. Children who have frequent headaches have inc rx for
impairment in academic and social functioning. Nonpharmacologic therapies, such as rest, relaxation techniques, and
removal of stress from the environment, can be effective once the stress is identified.

• Know the elements of history that characterize a headache due to increased intracranial pressure-
2006:183
• Know the signs and symptoms of a headache that indicate a need for follow-up with magnetic
response imaging or CT scan- 2006:7- Recent onset of severe headache, Change in type of h/a,
Neurologic dysfunction- Focal findings, Inc. ICP, Altered MSE, Coexistence of Seizures
• Know the values and limitations of ancillary neurodiagnostic tests in the evaluation of a headache-
2006:7
• Plan the abortive treatment of an acute migraine 2007;28:43 2007:38
NSAIDs, Sumatriptan, Amitryptiline, Ergot. Get good sleep, hydration, Avoid triggers (caffeine, citrus,
chocolate, cheese)
Preventive Meds: Topamax, Valproate, Gabapentin, Amitryptiline, NSAID, Verapamil, Propranolol
• Plan the treatment of a headache due to stress/tension/emotion 2007;28:43 2007:38

• Recognize the potential complications of using narcotics, sedatives, and nonsteroidal anti-
inflammatory drugs to treat a chronic or recurrent headache 2006:38
Complications: Analgesic Overuse Headache: >5times/wk use Meds. MCC ASA, Ibuprofen, Tylenol.
2. Altered level of consciousness
• Know to measure ammonia concentration and organic acid concentrations in neonatal coma-
2007:211
• Know the common causes of an altered level of consciousness 2006;27:331 2009:217
Causes of Altered MSE: AEIOU TIPS
A - alcohol, anoxia E – epilepsy I - insulin (diabetes), ingestions O - overdose
U - uremia, underdose T- trauma I – infection P – psychiatric S - stroke
(cardiovascular)

Clonidine overdose- neurologic – dry muc memb, pupil constrict, and respiratory depression, hypotonia,
bradycardia, hypotension, and cardiac arrhythmias. Treatment is supportive- place in ICU, ECG.

A sudden onset over minutes in a previously healthy individual suggests trauma or a cerebrovascular
accident; effects of toxic ingestions may present over several hours; and a more gradual onset
suggests infection, metabolic disease, or a space-occupying lesion. Vital signs, level of consciousness,
pupil size and reaction, motor responses, and skin findings crucial in narrowing potential causes and
determining appropriate diagnostic studies.

• Plan the initial phase of evaluation for an altered level of consciousness- 2006;27:331 2009:233
• Recognize that disorders of metabolism, liver, kidneys, lungs or heart can be manifested as
encephalopathy 2006;27:29; 331
Encephalopathy - characterized by a deterioration in mental status, which can range from a subtle change in
attention to an acute confusional state to a marked change in arousal.
MCC of acute encephalopathy in children is CNS infection. Also important to consider disturbances in systemic
metabolism. The first sign of a serious compromise in perfusion or oxygenation is a change in mental status. //
Ddx includes systemic causes, such as electrolyte disturbances, toxic ingestion, uremia, hypertension, and hepatic
failure. More subtle disorders include indolent infxns, postinfectious processes, rheumatologic conditions, and inborn
errors of metabolism. Importance of infectious, structural, and vascular abnormalities should be appreciated.

• Know which ingestions are likely to result in neurologic toxicity 2006;27:331


• In a patient with an altered level of consciousness, know the items of importance in the history of
ingestion 2006;27:29; 331

 Avner JR. Altered states of consciousness. Pediatr Rev. 2006:27:331-338.- (Read)


3. Ataxia
• Know the common causes of acute ataxia 2007:54
• Know the common causes of vertigo 2007:70
• Know how to evaluate a child with ataxia 2006:39 2010:214

Ataxia - incoordination of motor actions due to disease or disorders of the cerebellum or its connections. Progressive
ataxias can be categorized as acute/subacute, episodic/paroxysmal, and chronic progressive.

When assessing children who have progressive incoordination, four key steps can narrow the ddx:

1. Localization of the problem to the cerebellum alone or cerebellum with other levels of the nervous system (brain,
basal ganglia, brainstem, spinal cord, and nerves). i.e. bilateral symptoms with completely normal mental status and
no somnolence described for the boy in the vignette makes a cerebral cause less likely.

2. Clarification of the time course, such as chronic over a specific time period or intermittent worsening.

3. Identification of pattern of inheritance. Absence of an autosomal dominant disease pattern reduces the
likelihood of a number of spinocerebellar ataxias.

4. Identification, if possible, of a highly characteristic phenotype. For example, are there suggestive skin or eye
findings? For poss. eventual genetic followup.

Chronic ataxias in children are rare, but the most common diagnoses are Friedreich ataxia and ataxia telangiectasia
(both autosomal recessive) // and spinocerebellar ataxias 1, 2, 3, and 7 (all autosomal dominant). Friedreich
ataxia, can be confirmed genetically by identifying a more than 90 GAA repeat expansion in the Frataxin gene. A
recent clinical trial showed high-dose idebenone to be a beneficial treatment.

Ataxia telangiectasia usually presents earlier than 12 years of age with chorea/athetosis or ataxia, and although
neurologic symptoms precede dermatologic ones, by this age, the ocular telangiectasias (Item C214) should be
apparent.
Hydrocephalus and Dandy Walker syndrome both can cause chronic gait problems. Progressive hydrocephalus should
cause headaches and distal hyperreflexia, whereas Dandy Walker syndrome should not be progressive or be
associated with distal wasting.
*
Consider MRI in young children for r/o cerebellar and brainstem neoplasms such as astrocytomas, pontine gliomas,
primitive neuroectodermal tumors (medulloblastomas), or ependymomas.

• Know the prognosis of childhood acute cerebellar ataxia 2010:118

Cerebellar signs: nystagmus, tremor on hand activation and finger-to-nose testing, trunk bobbing, broad-based gait.
Subacute onset of symptoms suggests acute cerebellar ataxia, which usually is acquired after infxn or immunization.

*For children with acute ataxia, ddx includes neuroblastoma presenting as opsoclonus myoclonus ataxia syndrome..

Ryan MM, Engle EC. Acute ataxia in childhood. J Child Neurol. 2003;18:309-316
4. Movement disorders (involuntary, paroxysmal)
• Know the effective drugs for controlling chorea 2010:134
Chorea: restless, continuous, involuntary movements that are irregular in direction and amplitude
"milk-maid's grip" and "darting tongue" signs, simple motor command cannot be maintained due to choreic intrusions
or involuntary relaxations (Item C134).
MCC chorea: poststreptococcal, immune-mediated, aka, Sydenham chorea. Dx: antistreptococcal ab’s:
antistreptolysin O and anti-DNAse B.
Ddx: chorea associated with SLE, antiphospholipid antibody syndrome, hyperthyroidism.

Haldol- dopamine blocking agent. high-potency, but high incidence of adverse effects. Sydenham chorea is self-
limited and a low dose is generally helpful. Other antipsychotics (fluphenazine or risperidone) or dopamine-depleting
agents (tetrabenazine) also may be used.

* Trihexyphenidyl is an anticholinergic medication that may reduce dystonia, but it tends to worsen the symptoms of
Sydenham chorea.

• Differentiate between tic disorder and Tourette syndrome 2006:27


• Know the drugs useful in the treatment of Tourette syndrome 2008:103
• Know which drugs can cause movement disorders 2007:246
• Know that Tourette syndrome is associated with behavioral difficulties, learning disabilities, and
attention deficit disorder 2006:44
5. Increased intracranial pressure
• Know to exclude a mass lesion by brain imaging in the presence of increased intracranial pressure
2007;28:e77 2007:86
• Know the common causes of pseudotumor cerebri 2006:55
• Recognize the need to measure the opening pressure at lumbar puncture 2006:55
• Know the signs and symptoms of increased intracranial pressure in infants 2007;28:e77 2007:86
• Know the signs and symptoms of increased intracranial pressure in children 2007;28:343; e77
2007:86
• Know the contraindications of immediate examination of the cerebrospinal fluid 2008;29:417
2006:64
Focal neuro signs, inc ICP signs- i.e. vitals, vomiting, if pt is unstable
• Plan the medical management of a patient who has increased intracranial pressure 2008;29:417
Mannitol, Hyperventilate, Hypertonic saline, Ventriculostomy, ICP monitor
6. Weakness and hypotonia
• Distinguish among acute and chronic causes of weakness 2007:102
• Know the benefits and limitations of ancillary neurodiagnostic tests in the evaluation of weakness
(eg, serum creatine kinase activity, electromyography 2007:102
• Distinguish between central and peripheral nervous system causes of hypotonia
2008;29:243 2007:118
• Know the differential diagnosis of hypotonia in infants 2008;29:243 2010:150 2007:118
• Know how to evaluate hypotonia in infants 2007:118

Hypotonia, a decrease in the muscle's postural tone, can result from a wide range of disorders of the brain, spinal
cord (spinal muscular atrophies, spinal cord injury), nerves (hereditary motor-sensory neuropathies, congenital
hypomyelinating neuropathy), neuromuscular transmission (infantile botulism, myasthenia gravis), or muscles
(congenital myopathies and muscular dystrophies, metabolic myopathies such as acid maltase deficiency).

Table. Selected Neurologic Causes of Hypotonia


Central Nervous System
• Brain
–Injury: Hypotonic cerebral palsy
–Chromosomal: Angelman syndrome, Prader-Willi syndrome, Rett syndrome, trisomy 21
–Metabolic: Leukodystrophies (eg, metachromatic leukodystrophy*), peroxisomal disorders (eg, neonatal
adrenoleukodystrophy*), GM1 and GM2 gangliosidoses (eg, Tay-Sachs disease), organic acidemias
–Other: Cerebral malformations, benign central hypotonia, autism
• Spinal cord
–Injury: Hypoxic-ischemic myelopathy, trauma
–Anterior horn cell: Spinal muscular atrophies

Peripheral Nervous System (Motor Unit)


• Nerves: Hereditary motor-sensory neuropathies, acute inflammatory demyelinating neuropathy (Guillain-Barré
syndrome), congenital hypomyelinating neuropathies
• Neuromuscular Transmission: Infantile botulism, myasthenic syndromes (eg, familial infantile myasthenia)
• Muscle: Muscular dystrophies (eg, Duchenne muscular dystrophy, Fukuyama congenital muscular dystrophy*),
congenital myopathies (eg, central core disease), metabolic myopathies (eg, acid maltase deficiency)
*
May have features of both central and peripheral nervous system involvement.

CNS causes of hypotonia and developmental delay include cerebral malformations; acquired cerebral injury that may
occur with hypoxia, ischemia, or infection; genetic disorders such as Prader-Willi syndrome; and several metabolic
disorders, including disorders of the peroxisomes, lysosomes, and mitochondria.
B. Infection
1. Meningitis
• Know the common acute complications of meningitis 2008;29:417 2007:76
• Know the etiologies of neonatal meningitis 2008;29:417 2009:242
Neonatal meningitis – risk factors prenatally (maternal fever, chorioamnionitis, prolonged rupture of
membranes) or postnatally (bacteremia, urinary tract infection, or respiratory distress); attendant signs of
apnea, lethargy, / acidosis, and hypoglycemia.
PE: bulging or tense anterior fontanelle, irritability, fever, emesis,/ coma, and seizures; change in neuromotor tone
CSF values in bacterial meningitis- elevated protein > 100; WBC > 100; Gluc < 50% of serum

Neonatal enteroviral infection may follow maternal infection, - spring and summer. have sepsis syndrome,
coagulopathy, and meningitis. Hepatopathy and respiratory sx

The mortality rate of neonatal bacterial meningitis is 10%, but both mortality and late morbidity vary with the cause
of meningitis. MCC group B Streptococcus, Escherichia coli, other gram-negative rods, and Listeria. Although
neonatal GBS is common, it is associated with later onset (after 7 days), and a shocklike state.
[Herpes simplex viral meningoencephalitis is associated with a hemorrhagic pleocytosis, apnea,
seizures, coagulopathy, and hepatic transaminase values generally greater than 1,000 U/L.]

• Know the etiologies of meningitis in children 2008;29:417 2007;28:e77


• Know the causes of meningitis when no bacteria are isolated (eg, partially treated, parameningeal
focus, Borrelia, spirochete, M. tuberculosis) 2008;29:417
• Distinguish among cerebrospinal fluid findings in bacterial, fungal, and viral meningitis
2008;29:417
Glucose (mg/dL) Protein (g/dL) White Blood Cell
(mmol/L) (g/L) (x103/mcL) (x109/L) Differential Count Gram stain
Healthy newborn 30 to 120 30 to 150 <0.03 No PMNs Negative
Healthy child 40 to 80 20 to 40 <0.01 No PMNs Negative
Bacterial meningitis <1/2 serum Often >100 >1.0 >50% PMNs Often >90%
<10
Enteroviral >1/2 serum 40 to 60 0.05 to 0.5 >50% PMNs early (<48 h) <50% Negative
meningitis PMNs later (>48 h)
Lyme meningitis >1/2 serum 0.05 to 0.5 Predominance of lymphocytes and Negative
monocytes
Tuberculous <1/2 serum Often >100 0.05 to 0.5 Lymphocyte predominance Negative
meningitis <10

• Know the clinical manifestations of bacterial meningitis 2008;29:417


• Know the differential diagnosis of fever and petechiae/purpura in bacterial meningitis 2008;29:417
• Know the clinical manifestations of aseptic meningitis 2008;29:417
• Know the laboratory diagnosis of aseptic meningitis 2008;29:417
• Know how to manage subdural effusion associated with bacterial meningitis 2008;29:417
• Know the acceptable treatments of meningitis 2008;29:417
• Know the management of cerebral edema in meningitis 2008;29:417
• Know the importance of monitoring fluid balance and electrolyte concentrations in meningitis
2008;29:417
• Know the indications for diagnostic imaging in patients with meningitis 2008;29:417
• Recognize the need for auditory testing following meningitis 2008;29:417
• Know the potential long-term sequelae of meningitis 2008;29:417

For infants whose CSF is suspicious for bacterial meningitis, ampicillin (300 mg/kg per day divided every 6 hours)
and cefotaxime (200 to 300 mg/kg per day divided every 6 hours) is appropriate. If the child is younger than 4 to
6 weeks of age, acyclovir (60 mg/kg per day divided every 8 hours) should be added if HSV infection is a concern.
In the young infant, if the Gram stain suggests pneumococcus, vancomycin (60 mg/kg per day given every 6
hours) should be added.

For children older than 2 months, vancomycin (60 mg/kg per day divided every 6 hours) plus
ceftriaxone (100 mg/kg per day given in one dose or divided into two doses) or cefotaxime
(200 to 300 mg/kg per day divided every 6 hours) should be used for empiric coverage. / An
alternative therapy for children who have had anaphylactic reactions to penicillin or cephalosporins is
a carbapenem or a quinolone in addition to vancomycin
Appropriate parenteral antibiotics should be continued for 7 days for meningococcal meningitis
and 14 days for Listeria, GBS, and pneumococcal meningitis. Lyme meningitis typically is
treated with IV ceftriaxone (50 to 75 mg/kg per day given once daily) for 14 to 28 days.
Meningitis caused by gram-negative enteric bacilli requires a longer duration of therapy,
generally a minimum of 21 days.
Neonatal HSV CNS infection typically is treated w IV acyclovir (60 mg/kg per day divided every 8
hours) for 21 days

2.Encephalitis
• Know the common causes of encephalitis 2005;26:347 2006:231
• Recognize the signs and symptoms of herpes encephalitis 2005;26:347 2006:199
• Recognize the cerebrospinal fluid findings in herpes encephalitis 2005;26:347
• Know the clinical symptoms of encephalitis 2005;26:347
• Know the role of neurodiagnostic testing in the evaluation of a child with encephalitis 2005;26:347
2008:119
• Know the appropriate microbiologic, serologic, and molecular diagnostic tests in a child with
encephalitis 2005;26:347
• Know how to manage encephalitis 2005;26:347
• Know the common sequelae of encephalitis 2005;26:347

Encephalitis commonly due to an arbovirus or herpes simplex infection. Arbovirus more likely to occur in summer
because arbovirus is transmitted to humans by mosquitoes. On MRI may see subtle thickening. HSV results in
necrotizing focal encephalitis in the temporal lobes. In both, focal infection and inflammation in the brain's cortex can
lead to seizures. In the case of herpes, outside the neonatal period, the temporal lobe infection occurs because of
reactivation of the HSV in the trigeminal nerve and spread directly into adjacent brain tissue.

Virus also can enter the brain via the blood, in which case generalized encephalitis is more likely. Examples include
HSV in the neonatal period. Other viruses prone to cause encephalitis include measles, HIV, rabies and CMV.

Ingestion of the eggs of Taenia solium (pork tapeworm) can lead to neurocysticercosis. This can present with focal-
onset seizures, but affected patients usually do not appear ill, and CT/MRI reveal cysts (Item C166B).
Borrelia burgdorferi is transmitted by ticks, and early infection presents with the characteristic rash (erythema
migrans). Late-stage disease presents with facial nerve palsies.
Listeria monocytogenes often presents with brainstem encephalitis.

American Board of Pediatrics Content Specification:


Know the etiologic and therapeutic implications of focal versus generalized clinical manifestations of
encephalitis

3. Abscess
• Know the clinical manifestations of brain abscess 2009:6 -
Facial pain or relatively nonspecific pain, emotional problems, Confusion , psychomotor retardation - frontal lobe
symptoms, inc ICP- headache on awakening , vomiting. Brain abscesses often present only with nonspecific pain and
not with fever.

References:
• Goodkin HP, Harper MB, Pomeroy SL. Intracerebral abscess in children: historical trends at Children's
Hospital Boston. Pediatrics. 2004;113:1765-1770.
• Haslam RHA. Brain abscess. In: Kliegman RM, Behrman RE, Jenson HB, Stanton BF, eds. Nelson Textbook
of Pediatrics. 18th ed. Philadelphia, Pa: Saunders Elsevier; 2007:2524-2525

• Know that many brain abscesses contain multiple organisms, including anaerobes 2006:111
• Know that neuroimaging studies should be done before examination of the cerebrospinal fluid in
suspected brain abscess 2006:167
• Understand the imaging techniques for diagnosing brain abscess 2007;28:305
Head CT with Contrast is recommended because of the insidious onset, which could indicate
either a neoplasm or infectious process. Intravenous contrast increases the diagnostic yield of
imaging studies where either neoplasm or infection is suspected because both typically involve some
degradation of the blood-brain barrier or hypervascularity, resulting in contrast
enhancement at the site of the lesion.

• Understand the treatment of brain abscess 2008:76:


initial antimicrobial therapy should be broad, such as a combination regimen of vancomycin +
metronidazole + ceftriaxone. Vancomycin provides coverage for methicillin-susceptible and
-resistant Staphylococcus aureus as well as other aerobic gram-positive organisms (eg, streptococcal
species). Metronidazole penetrates the blood-brain barrier well whether administered intravenously or
orally and covers a large number of anaerobes. Ceftriaxone provides not only aerobic gram-positive
coverage (eg, streptococcal species), but also addresses gram-negative organisms (eg, Haemophilus
influenzae). The duration of antimicrobial therapy is usually 4 to 6 weeks.
• Know the propensity for brain abscess to complicate cyanotic heart disease, sinusitis, and
pulmonary disease 2006:127

4. Myelitis
• Plan the evaluation of a patient in whom post-infectious myelitis is suspected 2006;27:463 2006:71
C. Degenerative conditions
1. Signs and symptoms
• Recognize the historical features indicative of a degenerative CNS disorder 2005;26:218 2008:135

Neurodegenerative disorders: chronic and progressive, featuring selective and symmetric loss of neurons in sensory,
motor, or cognitive areas. Can affect primarily white matter (leukoencephalopathies) or gray matter. Sx of d/d: loss
of speech, hearing, vision, strength, coordination, intelligence, or memory. Seizures, feeding problems. MRI -
symmetric atrophy or demyelination of specific brain regions.

Ddx : poorly controlled seizure disorders, congenital and chronic infection (HIV), chromosomal anomalies,
hypothyroidism, structural or mass lesions of the brain, subacute sclerosing panencephalitis, and inhaled solvent
abuse (glue sniffing).

Incidence and Cause :The genetic neurodegenerative disorders are inherited as autosomal recessive, X-linked
recessive, or mitochondrial disorders. Single-gene defects cause these diseases. Those appearing primarily in infancy
and childhood include the leukodystrophies (X-ALD, metachromatic leukodystrophy, Krabbe disease [globoid cell
leukodystrophy], Zellweger syndrome, Canavan disease); mitochondrial disorders (Kearns-Sayre syndrome,
myoclonic epilepsy with ragged red fibers, and MELAS [syndrome of mitochondrial myopathy, encephalopathy, lactic
acidosis, and strokelike episodes]); diseases of the extrapyramidal nuclei (Leigh syndrome [infantile subacute
necrotizing encephalomyelopathy]); early-onset Alzheimer disease in patients who have Down syndrome; multiple
sclerosis; Tay-Sachs disease; and Niemann-Pick disease.

Clinical Features: Neurodegenerative diseases may present at any stage of life from the newborn period to
adulthood. Their presentation is highly variable. Symptom onset in the genetic disorders depends on the buildup of
toxic metabolites or lack of substrate. Illness, diet, and other environmental factors affect signs and symptoms,
which may wax and wane. Disorders presenting in neonates usually are severe, persistent, and rapidly progressive.
Patients having mild neurologic or behavioral symptoms may present subtly in childhood, adolescence, or adulthood.

The hallmark of a neurodegenerative condition is regression or an unexplained plateau in development; this state
may evolve rapidly in days or weeks or more insidiously over months to years. It is important to consider a
neurodegenerative disorder in any child whose mental retardation, seizures, or motor problems are unexplained.
Parental consanguinity or a family history of early infant death should raise suspicion. Many newborn metabolic
screening tests do not test for neurodegenerative disorders; thus, normal results may not be relevant. Neonates often
present with signs similar to those of sepsis.

Management, Therapy, and Prognosis Initial care focuses on correcting metabolic derangements.
Definitive care involves restricting substrate or eliminating toxic metabolites. Enzyme replacement is available for
some disorders. The prognosis varies, depending on the underlying disorder. Parents and families of affected children
require genetic counseling.

• Recognize the signs and symptoms of degenerative CNS disorders 2005;26:218 2008:135
• Know the clinical presentation and course of Rett syndrome 2008;29:243 2007:11 2006:215

The hallmark of degenerative disorders of the CNS is the progressive loss of previously acquired abilities. In infants
and young children, a deceleration in the rate of development is often the first presentation: the child falls
progressively behind other children and only subsequently loses previously acquired milestones. When the declining
developmental quotient is not due to an extrinsic agent or event or to secondary involvement of the CNS by a
generalized systemic disease, neurodegenerative disease becomes a consideration.

Most degenerative CNS disorders can be divided clinically into three groups: gray-matter diseases, white-matter
diseases, and system diseases. The gray-matter diseases, which primarily involve the neurons, occur with or without
histologic evidence of storage of abnormal metabolic products. They lead to neuronal death and secondary
degeneration of axons. // In the white-matter diseases, myelin is disrupted, either by the destruction of normal myelin
or by the production of biochemically abnormal myelin. The system diseases are a heterogeneous group of conditions
involving progressive degeneration of anatomically defined systems, such as the dorsal columns, pyramidal tracts, or
cerebellar nuclei. Typically, both neurons and myelin are destroyed in these disorders.

The first clinical task in evaluating a child for any neurodegenerative disorder is to document that he or she has lost
previously acquired milestones or has a decelerating developmental quotient. In either instance, the most important
diagnostic tool is repeated developmental evaluations.

The major clinical features that differentiate gray-matter from white-matter diseases are related directly to the
functional roles of the neurons and myelin. Neuronal involvement in gray-matter disease leads to the early onset of
dementia, the progressive loss of cognitive abilities, and seizures that frequently are myoclonic. The basal ganglia and
cerebellar nuclei are collections of neurons, so extrapyramidal and cerebellar signs, such as ataxia, are common.
Ganglion cells of the retina are affected in many of the gray-matter disorders, producing pigmentary degeneration of
the retina. Abnormal storage of lipid in retinal ganglion cells (eg, in Tay-Sachs disease) makes the perifoveal area
look gray and opaque. The fovea, which contains no ganglion cells, appears red by contrast, producing the clinically
detected "cherry red spot."

The majority of degenerative diseases of white matter stem from biochemical defects resulting in abnormal myelin
that breaks down rapidly. These "dysmyelinating" disorders are called leukodystrophies. Clinically, the earliest sign of
most white-matter degenerations is spasticity. Dementia and seizures can occur, but usually later in the clinical
course. Involvement of cerebellar pathways can cause ataxia. Rather than the cherry-red spot of gray-matter disease,
optic atrophy is the most characteristic ocular change seen in white-matter disease. Some patients even have cortical
blindness from demyelination of the optic pathways in the cerebral hemispheres.
*****
Rett syndrome - stereotypic hand movements, followed by seizures and dementia. RS affects girls,
exclusively. Although most cases are sporadic, the disease has long been suspected of having a genetic basis. Genetic
advances suggest that the gene for RS is located on the distal arm of the X chromosome.

Stage 1 (birth to 18 mo): Deceleration of head growth, beginning between 2 and 4 months of age, results in an
acquired microcephaly. / Babies show decreased interest in their environment and become markedly hypotonic.

Stage 2 ( 1 to 2 y): The loss of both expressive language and gross motor milestones, and the onset of abnormal
hand movements, seizures, irritability, and insomnia are typical of progressing RS.

Stage 3 (2 to 10 y): Severe mental retardation, seizures, and persistent stereotypic hand wringing movements
characterize the childhood phase of RS. Most patients develop ataxic and wide-based gaits, but many affected girls
never walk. Tremulousness is a common finding, as are breath-holding spells.

Stage 4 (older than 10 y): As girls who have RS approach adolescence, they develop progressive scoliosis, muscle
wasting, and can become wheelchair-bound. By the early teenage years, they reach a plateau in their neurologic
regression, but death usually occurs during late adolescence from infection or cardiac arrhythmia.

Neurodegen dis ddx: hydrocephalus, hypothyroidism, mass and structural lesions of the brain, chromosomal defects,
poorly controlled seizures, environmental deprivation, subdural hematomas, and congenital and chronic infections,
such as human immunodeficiency virus. Neurodegenerative disorders, however, are predominantly hereditary.
Although the biochemical basis of many neurodegenerative disorders is not fully understood, a precise biochemical
diagnosis always should be sought to make possible an accurate discussion of prognosis, to provide counseling, and
sometimes to administer specific treatment.

The availability of molecular genetic testing has improved the accuracy of diagnosis in symptomatic patients,
prenatally when genetic risk has been identified, and for carrier testing. Many neurodegenerative disorders can be
diagnosed by biochemical testing of blood or urine without invasive procedures to obtain tissue for direct
examination. Even so, a diagnosis of an altered enzyme disease should be confirmed histologically.
In general, treatment is available for only a few of the neurodegenerative diseases. Unfortunately, most remain
invariably fatal by late childhood. Dietary therapy, restriction of a substrate or substrate precursor, modification of
enzyme activity with cofactor vitamin therapy, and enzyme replacement all have been attempted with only limited
success. Hope for more effective treatment comes from developments in molecular genetics that may allow the repair
of specific point mutations in the human genome.

**

2. Diagnosis
• Understand the initial evaluation of a patient with suspected CNS degenerative disease- Rpt
developmental exams, MRI if neuro deficits or to eval gray vs white dis

D. Developmental malformation, static neurologic deficit


1. Malformations
• Know that a myelomeningocele is usually associated with hydrocephalus 2009;30:15 2005;26:50,
PIR Nov 2010
• Recognize the most common orthopedic problems associated with a myelomeningocele and their
relative significance 2005;26:50
• Understand the evaluation and fundamental long-term management of neurogenic bladder
2005;26:50
• Know the fundamental long-term management of a neurogenic bowel 2005;26:50
• Know the clinical and radiographic features and prognosis of spina bifida occulta 2005;26:50
• Know the differential diagnosis of acute neurologic deterioration in a child with myelomeningocele
2005;26:50
• Identify the clinical manifestations and plan the diagnostic evaluation of spinal dysraphism
2007:134
• Recognize the clinical features of hydrocephalus 2009:22

Headache, gait disturbance (toe-walking), acquired ocular misalignment (right eye cannot move to the
right) raises concerns for both hydrocephalus and a brainstem lesion.

Head CT scan is adequate to rule out hydrocephalus that requires emergent neurosurgical
consultation. Unfortunately, a common cause for hydrocephalus in a child of this age is a brainstem or
cerebellar neoplasm such as an astrocytoma, glioma, medulloblastoma, or ependymoma. When these
tumors enlarge in the posterior fossa, they can obstruct the flow of cerebrospinal fluid and cause
acute hydrocephalus, which is a neurosurgical emergency.)

• Recognize the signs and symptoms of shunt malfunction in hydrocephalus 2006:87


Obstruction (vomiting, altered MSE), Infection (fever)
• Know the differential diagnosis of microcephaly 2009:182
Dx: serial Head circumferences.. do MRI

Suboptimal head growth during the first year and microcephaly at 1 year of age are associated with subnormal
developmental outcomes in children who had hypoxic-ischemic injuries at term birth.

Head ultrasonography: alternative to MRI – No sedation. lower resolution for peripheral gray matter and deep
brainstem structures. Better choice for dx macrocephaly b/c diagnoses hydrocephalus.

High-resolution karyotyping may be reasonable. However, this evaluation can be deferred until after additional
information about brain structure has been obtained. For example, if there is evidence of a stroke, hypoxic-ischemic
injury, or a congenital infection, karyotyping may not be needed.

Three-dimensional computed tomography scan is used to diagnose craniosynostosis. Craniosynostosis usually


presents with an unusual head shape due to closure of one or more skull sutures.
2. Cerebral palsy
• Know that birth trauma and obstetric complications are not the leading causes of cerebral palsy
2006:60
• Recognize the clinical features of cerebral palsy, including classifications 2006:076
• Know the risk factors associated with cerebral palsy 2006:60
• Know the disabilities associated with cerebral palsy: cognitive, visual, communication, auditory,
motor, seizure activity, behavioral, oral function, nutrition 2006:76
• Know the principles of management for children with cerebral palsy (eg, feeding, spasticity,
mobility, activities of daily living, education) 2007:27

E. Seizures

 A seizure is the clinical manifestation of an abnormal, excessive electrical discharge of


nerve cells
 Epilepsy: a tendency toward recurrent seizures that are not a result of systemic or
neurological insults
 First seizure may be: Caused by an acute illness, Metabolic derangement , Infectious disorder

ILAE Classification of Seizures


I. Partial (Focal, Local) Seizures (involves part of the brain)
○ A. Simple Partial (consciousness retained). No postictal change in sensorium
B. Complex Partial (consciousness impaired)
C. Partial evolving to Generalized Tonic Clonic
II. Generalized Seizures (involves whole brain, loss of consciousness)
A. Absence
B. Myoclonic
C. Clonic
D. Tonic
E. Generalized Tonic-Clonic
F. Atonic
G. Infantile spasms
III. Unclassified Epileptic Seizures

 Complex means alteration in consciousness


 Involves facial movements (chewing, sucking, swallowing)
 Can appear to be responding to auditory or visual hallucinations
 May have automatism, movements or vocalizations such as moaning or simple talking
 Patients may complain of aura
Children unaware of seizure
Children can be postictal after

Generalized Seizures -non-focal seizures (involves whole brain, loss of consciousness)

Absence: Brief episodes where child stares into space with no awareness of their environment, minor
motor manifestations such as blinking and lip smacking. EEG described as a “3 per second spike
and wave”.

Seizure Features Generalized Partial


Aura Not Present Present
Prodrome Occasionally Occasionally
LOC Present Present
Automatisms Not usually Present
Prolonged Postictal state Not usually Present
GTC Present Present
Focal Motor Tonic or Clonic seizures Not usually Present
Family History Possible Not usually
History of CNS infection, significant Not usually Possible
head trauma, febrile seizure, CNS
tumor, vascular malformation, stroke
Neuroimaging Usually normal Possibly abnormal
EEG Generalized Focal Discharges
discharges
Neuro Exam Usually normal Possibly abnormal

**

1. General
• Recognize the metabolic causes of seizures 2007;28:363
• Know which drugs may precipitate or exacerbate seizures 2007;28:363 2006;27:283 2009:198
medications used in certain disciplines, most commonly oncology and psychiatry, may precipitate
them. Patients receiving cyclosporine or intrathecal methotrexate may develop diffuse or
predominantly occipital white matter changes. With cyclosporine, in addition to seizures, occipital
blindness or strokelike weakness may occur. The changes seen on magnetic resonance imaging can
be reversible. Other medications that may cause or exacerbate seizures in children include isoniazid, /
theophylline, cocaine, psychostimulants, bupropion, / insulin, and oral hypoglycemic agents.

• Know the most common causes of acute seizures 2007;28:263 2008:151


• Distinguish among epileptic seizures and paraxysmal non-epileptic events (eg, breath-holding
spells, tics, self-stimulation, syncope, gastroesophageal reflux, psychogenic seizures, sleep
disturbances) 2007;28:363 2006;27:e42
• Recognize the factors associated with an increased risk of seizure disorder 2007;28:363
• Know the etiologic and therapeutic implications of partial versus generalized seizures 2005;26:347
2009:166
• Know how to manage a child following a first seizure 2007;28:363 2008:167
• Know how to manage a child with recurring seizures 2007;28:363
• Formulate a management plan for a patient with psychogenic seizures 2007;28:363
• Know that drug selection is based on seizure type 2007;28:363
• Understand the indications for discontinuing anticonvulsant therapy 2007;28:363 2006:103
• Know the indications for initiating anticonvulsant therapy 2007;28:363
• Know the relationship between etiology and prognosis in seizures 2007;28:363 2008:182
• Know the side effects and toxicities of anticonvulsants 2007;28:363 2009:214
Antiseizure medications can cause a wide variety of adverse effects, including central nervous system
effects such as sedation, dizziness, personality changes, and occasionally, worsening of seizures.
Adverse effects of antiseizure medications may occur outside the central nervous system as well.
Bone marrow suppression and liver toxicity are among the more serious possible systemic effects.
Rashes are common in children and can occur as an adverse effect of most antiseizure medications.
Such drug rashes tend to occur during the first month of use, as described for the girl in the vignette.
Rapid assessment of the child is important because in some cases, the rash may be a harbinger of a
more global, serious allergic reaction or of Stevens-Johnson syndrome (erythema multiforme major).

• Know the value, limitations, and timing of serum drug concentration determinations during the
management of seizures 2007;28:363
• Know the laboratory abnormalities caused by anticonvulsants 2007;28:363
• Know the interactions of anticonvulsants with other drugs, including other anticonvulsants
2007;28:363
• Understand the cognitive/behavioral consequences of treatment with anticonvulsants 2007;28:363
• Understand the cognitive/behavioral problems associated with seizure disorders- 2007;28:363
• Provide appropriate counseling regarding activities and behavior of a child with a seizure disorder
(eg, athletics, school, driving, medications)- 2007;28:363
• Understand the psychosocial effects of epilepsy 2007;28:363
• Know the effects of epilepsy and anticonvulsant therapy on reproductive health (eg, contraception)
and the fetus- 2008:88

AED Efficacy Comments Side Effects


Carbamazepin Partial, General CP-450 inducer Hyponatremia
e
(400mg /day)
Ethosuximde Absence No CP-450 induction Anorexia
(250- or inhibition
500mg/day)
Gabapentin Partial Renal clearance Weight gain, myalgia
(300-
900mg/day)
Lamotrigine Partial, General Induced by PHY, Steven-Johnson, insomnia
(25-50mg/day) CBZ. Inhibited by
VPA
Oxcarbamaze Partial Prodrug of Hyponatremia
pine monohydroxy CBZ
(600mg/day)
Phenobarbitol Partial, General CP-450 inducer Porphyria exaserbation
(30-90mg/day)
Phenytoin Partial, General CP-450 inducer Osteomalacia, gingival disease
(300mg/day)
Tiagabine Partial No enzyme induction Seizures
(4mg/day) or inhibition
Topiramate Partial, General May increase PHY Renal stones, glaucoma,
(25- levels, no AED paraesthesias, weight loss, word
50mg/day) enzyme induction, finding problems
may alter efficacy of
OCP
Valproate General CP450 inhibitor Weight gain, hair loss, liver
(500- disease, tremor
1000mg/day)

2. Neonatal
Neonatal seizures tend to be focal, rather than generalized, and may be clonic (rhythmic jerking) or
tonic (sustained), but usually are not tonic-clonic. Focal clonic seizures may involve more than one
limb, but if jerking movements are bilateral, they are asynchronous.

3. Febrile
• Know the natural history of febrile seizures 2007;28:363
• Know the risk factors associated with febrile seizures related to later epilepsy 2007;28:405
2010:230 2006:166
Only a small percentage of children who have febrile seizures subsequently develop recurrent nonfebrile seizures or
epilepsy. Risk factors identified by the National Collaborative Perinatal Project include abnormal neurologic
development and a first febrile seizure that is complex. A family history of epilepsy may increase the risk. A family
history of febrile seizures and young age of onset increase the risk for future febrile seizures, but not for epilepsy.
• Know the diagnostic criteria for a febrile seizure 2007:28:363; 405
• Know the appropriate evaluation of a child with febrile seizures 2006:182
• Know the management of febrile seizures 2007;28:363; 405

4. Infantile spasms
• Recognize the characteristic clinical features of infantile spasms 2006;27:389 2009:230
• Know the prognosis for children with infantile spasms 2006;27:389
 West syndrome is triad of: Infantile spasms, Hypsarrhythmia on interictal EEG {in between
seizure episodes}, Mental retardation- Devel. Delay; poor prognosis

4-6 mo of age
myoclonic-like seizures that can be subtle, such as a head drop or more dramatic, such as a "clasp-knife" spasm of the
body with arm flexion or extension. clustering is common, particularly at times of sleep-wake transition. the ictal
spasms are accompanied by changes in cognitive and motor function, manifested by reduced social interaction and
activity
severely chaotic, cortical electrical pattern identified on EEG- "hypsarrhythmia." Outcome is almost uniformly poor
for infants who have pre-existing neurologic problems and develop infantile spasms. Neurologic outcomes sometimes
are good for those who have previously normal neurodevelopment.
tuberous sclerosis may present with infantile spasms- need good PE and neuroimaging
tx = ACTH, vigabatrin in Canada. May consider steroids/focal resection depending on prognosis

5. Absence epilepsies (petit mal)


• Understand the drugs used to treat absence epilepsy 2007;28:363
• Recognize the characteristics of absence epilepsy 2007;28:363
>5y/o, F>M, No aura or postictal, <30 s, come in clusters
Tx: Ethosuximide, Valproic acid
Prognosis: The younger the child, the more likely to respond to therapy and remain free of seizures
even after discontinuation

6. Complex partial seizures


• Recognize the clinical features of complex partial epilepsy 2007;28:363 2008:197
• Understand the drugs used to treat complex partial seizures 2007;28:363 2008:197-
Carbamazapene, Lamictal, Phenobarbital, Phenytoin, Topamax

7. Status epilepticus
• Know to measure serum glucose, electrolyte, calcium, and magnesium concentrations in a patient
with status epilepticus 2007;28:363 2010:102
• Know the medications that can be administered rectally to treat status epilepticus 2007;28:363
• Know the possible etiologies of status epilepticus: infection, toxin, electrolyte imbalance,
drug withdrawal 2007;28:363 2009:246

Sz ddx: Intracranial lesions (ischemic strokes, trauma, hemorrhages, neoplasms, or focal or generalized brain
infections), metabolic derangements (glucose or electrolytes).

If the seizure has a focal onset, there is a residual focal deficit, or the seizure is accompanied by fever and prolonged
confusion, imaging studies should be ordered emergently. For infants or for those children whose histories suggest
electrolyte disturbance, glucose and electrolyte testing can be helpful. Electroencephalography (EEG) may be helpful
if confusion is prolonged because occasionally the prolonged confusion may be due to nonconvulsive status
epilepticus. In this case, however, the child's postictal state is resolving. Therefore, EEG will not add anything useful
to initial medical decisions.

MCC Status epilepticus: In epilepsy: Noncompliance. Also consider Illicit drugs.

8. Epilepsy syndromes
 Epilepsy syndrome: A complex of signs and symptoms that define a unique epilepsy
condition. This must involve more than just the seizure type.
 Epileptic Partial Syndrome- Benign Rolandic Epilepsy
 Epileptic Generalized Syndrome- Juvenile Myoclonic Epilepsy

• Identify the clinical manifestations of rolandic epilepsy 2007;28:363 2010:102


• Understand the appropriate treatment of rolandic epilepsy 2007;28:363
Benign rolandic epilepsy: Most common form of epilepsy in childhood
 M>F. Onset 3-13 years (peak: 9-10 years); Recovery before 15-16 years
 Seizures: brief, self-limiting seizures involving the face, Unilateral sensory involvement
(involving one side of the tongue)
Inheritance is autosomal dominant. centrotemporal spikes on EEG.
No treatment needed because usually nocturnal seizures, infrequent, do not cause any problems the following day.
A Partial seizure- use medications such as carbamazepine, if any.

• Recognize the clinical manifestations of juvenile myoclonic epilepsy 2007;28:363

Juvenile myoclonic epilepsy - teenage years -1) myoclonic jerks, usually in the morning; 2) generalized tonic-clonic
seizures (gen. epilepsy) and 3) absence seizures.

 Age-related onset (usually 12-18)


 Condition usually persists throughout life
 Familial or sporadic
 Seizures predominantly shortly after awakening
 Seizures precipitated by sleep withdrawal, light sensitive
 Seizure types: myoclonic (100%), generalized tonic-clonic (90%), absence (30%)
 Jerky movements within couple of hours after awakening.
 Normal intelligence
 No loss of consciousness
 Treatment: Valproic Acid is treatment of choice
 Keppra, Topamax, Lamictal all other options
 Patients usually require lifelong treatment with anticonvulsants, but overall prognosis is
generally good.

9. Tuberous sclerosis
• Recognize the clinical manifestations of tuberous sclerosis, and manage appropriately 2009:38

seizures, hypotonia, hypopigmented macules, and MRI findings suggestive of tuberous sclerosis
complex (TSC) (Item C38A).

Diagnostic assessment should be directed toward confirming whether this child has TSC. In many
cases, this is a clinical diagnosis based on the characteristic findings of skin examination and the
cerebral complications. However, many other organs may be involved at presentation or during the
child's lifetime, including the eyes, kidneys, lungs, and heart. In the future, this child is at risk for
developmental learning difficulties, behavior problems that can include features of autistic spectrum
disorders, and malignancies. Given the autosomal dominant inheritance, proper management involves
assessment of the parents and genetic counseling. Genetic testing may have false-negative results
due to mosaicism, ie, some organs may be affected due to TSC1 or TSC2 mutations that are not
present in blood.

GTC: Avg last abt 5 min, postictal state, LOC, loss of bladder ctrl with full body tonic-clonic movement
Refer to slides 26-28 for GTC outline
 Drug of choice: Valproic acid
 Phenytoin and carbamazepine are other options
 Lamotrigine (Lamictal): Broad-spectrum anti-epileptic
 A potential side-effect is Stevens-Johnson Syndrome

 Myoclonic : (myo = muscle, clonic = jerk)


 Single, repetitive, involuntary contractions of an isolated group of muscles
 Key characteristic is loss of muscle tone and sudden brief shock-like muscle
twitches with child falling forward.
 No impairment of consciousness or memory

F. Cerebrovascular disease
1. Stroke
• Identify the clinical features of childhood stroke 2007;28:305 2006;27:271
• Know the causes of stroke in children 2007;28:305 2006:135

The Condition
Stroke has emerged as a relatively common but underrecognized cause of neurologic disability in children. An
incidence of 3 to 8 per 100,000 children per year suggests that ischemic strokes are more common than pediatric brain
tumors. Most ischemic strokes in children are arterial, but 20% occur as a result of cerebral sinovenous thrombosis.

The sudden onset of focal neurologic deficits in a child should be considered to represent a stroke until proven
otherwise. Children who suffer strokes usually present with abrupt onset of hemiparesis, but other focal deficits,
including hemisensory changes, visual loss or diplopia, loss or slurring of speech, and imbalance or incoordination,
should raise suspicion. Confounding signs and symptoms such as headaches, seizures, and altered consciousness are
common.

Table. Differential Diagnosis of Pediatric Stroke

• Meningitis, Encephalitis
• Demyelination
• Hypoglycemia
• Seizures or a postictal state
• Inborn errors of metabolism

• Migraine

A risk factor is identified in more than 70% of children afflicted with stroke, and many harbor multiple risks. Three
large categories of associations are cardiac disease, arteriopathies, and prothrombotic disorders. Complex congenital
heart lesions are associated commonly, with interventional or surgical procedures also increasing the risk.
Arteriopathies include arterial dissection, Moyamoya disease, and sickle cell disease as well as inflammatory
conditions related to infection (meningitis, varicella) or vasculitic conditions. Prothrombotic disorders include the
factor V Leiden mutation, elevated lipoprotein (a) concentrations, and deficiencies of protein C or S. A wide variety
of acute and chronic systemic illnesses may increase the risk of pediatric stroke.

Stroke is diagnosed by clinical and neuroimaging findings. CT can demonstrate AIS and rule out hemorrhage, but is
insensitive in the acute phase. MRI is the investigation of choice; diffusion-weighted MRI has revolutionized the early
diagnosis of cerebral ischemia. Angiography can be accomplished with MR or CT techniques, although conventional
angiography may be required for diagnoses such as dissection. Additional investigations such as echocardiography
and prothrombotic testing are used to eliminate other risk factors.

Management
A child suspected of having had a stroke should be seen urgently by a pediatric neurologist. Evidence-based
management is lacking, but recent consensus-based publications provide useful guidelines. The immediate
thrombolytic ("clot-busting") treatments proven in adult stroke remain unproven in children, but studies are underway.
Acute anticoagulation therapy with heparin or LMWH appears safe and may decrease the early progression of stroke
and multiple strokes observed in this patient. Published guidelines recommend either initial aspirin (ASA) therapy or
acute anticoagulation until investigations have delineated the cause for the stroke, followed by 3 to 6 months of
anticoagulation for arterial dissection or a presumed cardiac cause. Early management also should provide
supportive care to minimize secondary brain injury and should include maintenance of normal blood pressure, blood
glucose concentration, and temperature, along with aggressive treatment of infection and immediate treatment of
seizures.

Patients whose strokes are due to other or unknown causes may be maintained on long-term ASA (3 to 5 mg/kg per
day), which also is recommended for patients suffering dissection or cardiogenic stroke, following 3 to 6 months of
anticoagulation. Such secondary stroke prevention is important because the risk of ongoing recurrence of AIS,
particularly in the first 6 months, is 10% to 25%. Early and aggressive physical, occupational, and speech therapy is
essential.

Only about 33% of children will be neurologically normal after a stroke, and the mortality rate is 5% to 10%. Most
survivors live with moderate-to-severe disability. Motor deficits are most common, but other sequelae include
language disorders, cognitive and behavioral problems, movement disorders, headaches, and epilepsy. The burden of
illness is exacerbated because the morbidity of stroke in a child affects the entire family and lasts a lifetime.

Lessons for the Clinician


Acute onset of a focal neurologic deficit in a child is an emergency and should be considered a stroke until proven
otherwise. A high degree of clinical suspicion is required to improve recognition of pediatric stroke and avoid delays
in diagnosis. Because time-dependent treatments now are available and evolving, the importance of improving
awareness and diagnostic sensitivity cannot be overemphasized

2. Vascular anomalies
• Identify the clinical features of CNS arteriovenous malformations of childhood 2009:54

The most common cause of hemorrhagic stroke in children is vascular malformation, of which there are two types:
arteriovenous malformations (AVMs) and cavernous malformations. These can present in childhood with hemorrhage
that leads to headache and seizures.

AVM:
Cavernous:

G. Spinal cord disease


1. Signs and symptoms
• Recognize the clinical manifestations of an acute spinal cord lesion 2006;27:463
• Know the association between atlantoaxial instability in Down syndrome and potential neurologic
complications 2008:104
• Recognize the significance of bladder and bowel dysfunction in spinal cord disease2007;28:193
2008:39

2. Diagnosis
• Plan the initial neurodiagnostic evaluation in a patient with acute spinal cord dysfunction 2009:70

Acute spinal cord lesion: neurologic emergency. In some cases, emergency neurosurgery is needed for lesions causing
acute spinal cord compression, and the initial diagnostic emphasis should be directed toward identifying such lesions

Back pain, acute bilateral flaccid weakness, and sensory loss below the level of the lesion localize to the spinal cord.
Rapid onset of symptoms and the preservation of vibratory and proprioceptive sensation point to the anterior cord,
bilaterally, which is consistent with an anterior spinal artery stroke. Spinal cord strokes in children are uncommon but
can occur after aortic surgery or as a consequence of thrombotic disorders, infection, inflammatory diseases, or
trauma.

MRI = test of choice. This should aid in determining the specific location of the lesion and whether
emergency neurosurgery to decompress the spinal cord is needed.
H. Peripheral nerve and nerve roots
1. Guillain-Barre syndrome
• Know the presenting signs and symptoms of Guillain-Barre syndrome 2006;27:147 2008:212
• Know the risk factors that are associated with Guillain-Barre syndrome (eg, recent immunization,
varicella infection) 2006;27:147
• Know the differential diagnosis of Guillain-Barre syndrome 2006;27:147
• Know the expected results of laboratory procedures such as examination of the cerebrospinal fluid,
nerve conduction studies, and electromyography in Guillain-Barre syndrome 2006;27:147
• Know that cranial nerves may be affected in Guillain-Barre syndrome 2006;27:147
• Know that autonomic dysfunction in Guillain-Barre syndrome may be prominent and dangerous
2006;27:147
• Understand the treatment of Guillain-Barre syndrome (eg, IVIG, plasmapheresis) 2006;27:147
2010:198

Differential Diagnosis
The differential diagnosis of neuromuscular disorders includes CNS disease (meningitis, encephalopathy, neoplasm),
peripheral nervous system disorders (drug toxicities, GBS, tick paralysis, diphtheria), and neuromuscular
junction/muscle disorders (botulism, myasthenia gravis, spinal muscular atrophy, neuromuscular blocking agents,
acute inflammatory myopathies, metabolic myopathies). CNS disease ruled out by the lack of fever to indicate an
infection, normal CSF, and normal CT and MRI. PNS ruled out with no exposure to medications, insects, insecticides,
and negative urine toxicology.

Acute, rapidly progressive, ascending symmetric muscle weakness; lack of deep tendon reflexes; prodromal viral
illness (URI, Gastroenteritis), vaccines or surgery. Immune-mediated; PNS.

Dx: clinical , CSF, EMG.

Pathophysiology
it has been postulated that the patient is infected by a pathogen that shares similar antigenic sites as the host axon or
the peripheral nerve myelin, resulting in an autoimmune response. This reaction leads to multifocal areas of
inflammation, especially at the spinal roots and peripheral nerves, followed by demyelination from macrophages
invading the basement membranes of the Schwann cells. The demyelination leaves the axon exposed, resulting in
electrical nerve impulse defects and eventual conduction block and flaccid paralysis. The intense inflammation at the
ventral and dorsal roots leads to the breakdown of the blood-brain barrier and the transudation of the plasma protein
into the CSF. The viruses involved most commonly include cytomegalovirus, Epstein-Barr virus, other herpesviruses,
and human immunodeficiency virus. Bacterial agents include Campylobacter jejuni (most frequently involved),
typhoid, paratyphoid, Listeria, and Mycoplasma pneumoniae. Other antecedent events are surgery and vaccines.

Clinical Features
progressive, ascending weakness with symmetrically decreased DTRs. GBS occurs in all age groups, although it is
rare in infants, and develops over hours to weeks

flaccid weakness, ataxia, sensory disturbance, autonomic dysfunction, Cranial nerves affected in 33% of patients. The
severity of weakness in GBS ranges from mild to total paralysis. Sensory disturbance is frequent, with some patients
complaining of pain or paresthesias in the extremities, around the mouth, and in the back. Autonomic dysfunction may
present as intermittent tachycardia, bradycardia, hypertension, and orthostatic hypotension. Bladder and bowel
dysfunction are rare.

Miller-Fisher variant: cranial nerve involvement (ophthalmoplegia), areflexia, ataxia.

Diagnosis
CSF: 1 to 2 weeks into the illness, the CSF analysis reveals protein 80 - 200 mg/dL , cell count < 10 cells/mm3, with
predominantly monocytes ("cytoalbuminemic disassociation").
NCS: absent or reduced F waves (seen early in the disease), low-amplitude or absent sensory nerve action
potentials, and prolonged latencies. EMG may show a pattern consistent with acute muscle denervation.

Management
supportive care, with close monitoring for autonomic disturbance (the most common cause of death) and
detection of respiratory failure (the second most common cause of death). Patients who demonstrate rapidly
progressive weakness, signs of aspiration, respiratory distress, or autonomic instability require admission to an
intensive care unit. 20% require mechanical ventilation. Intensive airway care and chest physiotherapy are important
in preventing pneumonia.

IVIG and plasmapheresis shorten the duration and severity of illness. IVIG is given daily for 5 days. Plasmapheresis
generally is administered as four double-volume plasma exchanges on alternate days and is most effective if given
within 3 weeks of the onset of symptoms. The relative safety, ease, and comfort of administration tend to make IVIG
the preferred treatment choice. Physical therapy should be a part of every affected child’s treatment.

AKA AIDP (acute inflammatory demyelinating polyneuropathy) manifested by progressive flaccid paralysis. The ascending,
symmetric paralysis usually follows a nonspecific viral infection or vaccination by about 2 to 4 weeks. The weakness begins in the
lower extremities and may progressively affect the trunk, upper limbs, and bulbar muscles. GBS predominantly involves the motor
nerves and occasionally the autonomic (blood pressure and heart rate variability) and sensory nerves (paresthesias). Bulbar
involvement may occur in up to 50% of cases. Signs of bulbar weakness include poor mobility of the palate when saying "ah",
decreased or absent gag, slurred or nasal speech, and coughing or sputtering after swallowing. // Respiratory insufficiency may lead
to the need for mechanical ventilation in 20% of affected patients. Suggested indications for elective intubation include: vital
capacity less than 10 to 15 mL/kg, maximal inspiratory pressure less than -30 to -20 cm H2O, significant bulbar
dysfunction, or a continued rise in PaCO2 (Table 1).

Table 1: Ventilatory Management for GBS

Respiratory Pathophysiology Vital Capacity (mL/kg) Ventilatory Management

None 40 to 70 Observation

Poor cough and increased secretions ~30 Chest physical therapy

Sigh mechanism depressed: atelectasis and ~25 Incentive spirometry


hypoxemia

Sigh lost: atelectasis and shunting ~15 to 20 Consider NIV/intubate electively

Hypoventilation and hypercapnia ~10 to 15 Full ventilation

The lack of palate movement when asked to say "ah" and the nasal speech are suggestive of bulbar involvement.

Mechanisms for efficacy of IVIG include inhibition of complement binding, neutralization of selected cytokines, downregulation of
antibody production, and modulation of Fc-receptor mediated phagocytosis.

American Board of Pediatrics Content Specifications:

• Recognize the clinical signs of Guillain-Barré Syndrome (paresthesia in the arms and legs, proximal ascending
weakness and respiratory failure closely associated with the loss of gag reflex)
• Know the criteria for elective intubation in patients with Guillain-Barré Syndrome (e.g., loss of gag reflex, declining
respiratory function, pharyngeal dysfunction)

Lumbar puncture is the appropriate test for suspected Guillain-Barré syndrome (GBS), an acute inflammatory
demyelinating polyneuropathy. Typically, affected children present with weakness beginning in the proximal legs,
pain, and absent reflexes. A variant of GBS can involve predominantly the brainstem and cerebellum, but usually
weakness is more extensive than described for this child, and the weakness is not fatigable. Further, at this early stage,
the characteristic cerebrospinal fluid findings in GBS, normal cells and high protein, often have not emerged.
Therefore, lumbar puncture is not the preferred study in this setting. Cold caloric testing evaluates vestibular function.
This patient has no nystagmus and does not require this uncomfortable test. Neurophysiologic testing with auditory or
visual evoked potentials is not needed when no clinical evidence suggests that these systems are affected.

The evaluation of subacute generalized weakness is a medical emergency because symptoms may progress to
respiratory insufficiency or dysautonomia and death. The evaluation should be systematic, considering possible
causes at the level of brain, brainstem, spinal cord, anterior horn cell, root, nerve, junction, and muscle. The girl
described in the vignette has generalized weakness, with normal mentation and sparing, at present, of muscles of the
face and swallowing. Therefore, the problem is less likely to involve the brain or brainstem. Similarly, the absence of
sensory loss makes spinal cord involvement unlikely. The back pain and absent reflexes localize the problem to root
and nerves, and the classic cerebrospinal fluid findings (high protein concentration, normal cells) confirm the
diagnosis of acute inflammatory demyelinating polyneuropathy (AIDP), also known as Guillain-Barré syndrome
(GBS).

The treatment of choice for AIDP/GBS in adults and children is intravenous immune globulin, administered at 2 g/kg
total dose. Respiratory status, with forced vital capacity or negative inspiratory force, and cardiovascular status should
be monitored in the hospital due to the risk of death. Neuropathic pain also should be treated, and both gabapentin and
carbamazepine are effective.

2. Neuropathy
• Recognize the clinical manifestations of childhood peripheral neuropathy 2006:151
• Know the common causes of peripheral neuropathy in childhood (eg, hereditary sensory and motor
neuropathy) 2008:55

• Recognize the signs and symptoms of and plan treatment for Bell palsy 2007;28:465 2010:22

Ddx: Acute unilateral facial weakness: acute facial nerve palsy (ie, Bell palsy), a more rostral disease process of the
brainstem or brain (cerebrum, motor cortex) such as a stroke. The key diagnostic point for facial weakness is whether
the weakness involves the entire side of the face or the face below the forehead. A 7th nerve palsy affects all the
innervated muscles, weakening the entire hemi-face from forehead to chin. A lesion above the facial nerve nucleus
typically weakens the face below the forehead.
When the examination localizes a problem involving facial weakness to the brain or brainstem, brain magnetic
resonance imaging (MRI) or, if MRI is not available quickly, noncontrast head computed tomography should be
obtained, whereas no imaging is needed if below the area of the brainstem.

The AAN practice parameter states that oral steroids probably are beneficial and acyclovir possibly is beneficial for
treatment of facial nerve palsy. Many clinicians recommend administering a short course of oral prednisone for Bell
palsy.

3. Brachial plexus injuries at birth


• Recognize the clinical manifestations of neonatal brachial plexus injuries 2006;27:238 2008:193
• Know the management and prognosis of neonatal brachial plexus injuries 2006;27:238 2008:193
Intrapartum trauma to the brachial plexus encompasses a spectrum of injuries involving the lower cervical and upper
thoracic nerves (C5 through T1), which supply the plexus. These five spinal nerve roots combine to form the upper
(C5 through C6), middle (C7), and lower (C8 through T1) trunks of the plexus, and the peripheral nerves originating
from the plexus innervate the muscle groups of the shoulder, upper arm, forearm, wrist, and hand. The phrenic nerve,
comprised of fibers from C3 through C5, and the sympathetic fibers of T1 are affected commonly in brachial plexus
injuries, resulting, respectively, in ipsilateral diaphragmatic paralysis and Horner syndrome (miosis, ptosis,
anhidrosis of the affected side).

The incidence of brachial plexus injury is approximately 1 in 1,000 live births. Erb palsy, injury to the C5 through C7
spinal nerves, accounts for approximately 90% of cases. // Klumpke palsy, injury to the lower trunk (C8 through T1),
is rare and accounts for 1% of cases. Total plexus injury accounts for 10%. Bilateral injury is found in 10% to 20% of
cases, occurring almost exclusively in the setting of breech presentation.

Maternal, fetal, and parturitional factors can affect an infant’s risk for brachial plexus injury. Maternal risk factors
include uterine abnormalities, such as fibroids or a bicornuate uterus, and maternal diabetes. Fetal factors that may
predispose an infant to injury include macrosomia, transverse lie, poor tone, and neonatal depression. Among the
parturitional factors are abnormal presentations, dysfunctional labor, and the mechanical forces of labor. Although
the etiology of brachial plexus may be multifactorial, the pathogenesis of injury is believed to be from traction or
stretch injury to the plexus. The classic example is injury from extreme lateral flexion and traction of the head in the
attempt to deliver the shoulder during cephalic deliveries that involve shoulder dystocia. Conversely, traction can be
applied to the plexus via the shoulder in the process of delivering the head during breech deliveries. Abdominal wall
and intrauterine forces acting on a posterior shoulder pressed against the sacral promontory also have been implicated
in brachial plexus injuries.

There are four types of neuronal injury, and the severity of injury influences the likelihood of spontaneous recovery.
The most severe form of injury, associated with poor spontaneous recovery, involves avulsion of the nerve root from
the spinal cord, often with injury to the cord itself. Equally poor in prognosis is neurotmesis, axonal rupture with
disruption of the nerve sheath. Outcome is somewhat improved with axonotmesis, which involves axonal rupture with
the nerve sheath left intact. Fortunately, the most common form of injury, neuropraxia, involves damage to the nerve
sheath alone, resulting in a temporary conduction block. This type of injury is associated with complete spontaneous
recovery.

The clinical manifestations of brachial plexus injury often are recognized in the labor suite. Infants who have Erb
palsy have been described classically as having a "waiter’s tip" appearance. Weakness at the shoulder of abduction
and external rotation, at the elbow of flexion and supination, and at the wrist and fingers of extension results in
adduction and internal rotation of the shoulder, extension of the elbow, pronation of the forearm, and flexion of
the wrists and fingers. The biceps reflex is not present, and the Moro and tonic neck reflexes are asymmetric.

Klumpke palsy presents with weakness of the long flexors of the wrist and the intrinsic muscles of the hands,
resulting in an absent grasp reflex. The biceps reflex is intact.

A flaccid arm that has absent reflexes throughout indicates total plexus injury. An asymmetric Moro reflex
suggests a brachial plexus injury.

Management of an infant who has a brachial plexus injury requires gentle handling of the affected limb to avoid
additional trauma. Many affected infants have a torticollis, putting them at risk of developing plagiocephaly.
Radiographs of the clavicle and humerus should be obtained to exclude bony injury. The infant should be observed for
respiratory distress (potential diaphragmatic injury) and examined for signs of Horner syndrome. Magnetic
resonance imaging and electromyography have been used as adjunctive examinations, more in planning surgical
exploration than in routine diagnosis and management. Prolonged immobilization is not recommended. Generally,
infants should be referred to therapists for passive range of motion exercises beginning 7 to 10 days after birth.
Therapy should include passive range of motion of the neck and proximal arm.
The prognosis of brachial plexus injuries depends on the severity and extent of the lesion. Upper injuries, limited to
C5 through C6, have the best prognosis; // lower plexus and total plexus injuries have poor prognoses. Because Erb
palsy, involving only the upper plexus, is by far the most common injury, spontaneous recovery occurs in
approximately 90% of cases. Onset of recovery within 2 to 4 weeks is a favorable prognostic sign; flaccid paralysis of
the entire limb, especially with coexisting Horner syndrome or diaphragmatic paralysis, is an adverse prognostic
factor.

Follow-up care involves serial neurologic examinations. If antigravity movement of the affected muscle groups is
present by the end of the third month of age, the prognosis is excellent. Satisfactory, but imperfect recovery can be
expected if perceptible contractions are present by the end of the third month and antigravity movement is present by
the end of the fifth month. If no progress toward recovery is made within the first 2 to 3 months after birth, referral to
a specialized center for evaluation is warranted. If no improvement is detected between 3 and 6 months, the likelihood
of spontaneous recovery is grim, and surgical exploration may be considered. Overall, more than 90% of patients are
destined to attain complete recovery by 4 months of age. Although recovery of function is most rapid in the first few
postnatal months, improvement may continue for up to 1 year of age.

I. Neuromuscular junction
• Recognize that ticks may cause paralysis 2008:71
• Know the signs and symptoms of myasthenia gravis 2008:242
• Understand the laboratory and electrophysiologic studies to evaluate children with myasthenia
gravis 2009:102
• Understand the appropriate management for a patient with myasthenia gravis 2009:102

Fatigable weakness means that after a period of sustained use, muscle strength diminishes. Such a pattern is
characteristic of weakness due to pathology at the NMJ.

New-onset weakness mandates urgent evaluation and consideration of problems involving airway protection and
respiratory insufficiency, particularly if there is evidence of generalized weakness. If the pharyngeal muscles are
affected, speech sounds hyponasal. Negative inspiratory force or forced vital capacity should be documented
promptly because these findings determine whether the child should be transferred to an ICU for ventilatory
monitoring and support.

Ocular fatigability suggests a neuromuscular junction problem such as myasthenia gravis.

Dx: ("Tensilon® test"), acetylcholinesterase inhibitor edrophonium. Because the test can pose a risk of life-threatening
bradycardia, it should be undertaken by an experienced physician with atropine at the bedside. Neurologic
examination before and after administration ideally should be documented on video (Item C102).

Treatment - immunomodulation (eg, intravenous immunoglobulin or plasmapheresis) ; acetylcholinesterase inhibitors.


Imaging to assess for the presence of a thymoma also.

American Board of Pediatrics Content Specifications:

• Know the laboratory and electrophysiologic studies to evaluate children with myasthenia gravis
• Provide appropriate management for a patient with myasthenia gravis

J. Muscle diseases
1. Signs and symptoms
• Know the clinical features of dystrophinopathy (Duchenne/Becker 2006;27:83 2006:247muscular
dystrophy)
• Know that a Gowers sign indicates proximal muscle weakness 2006;27:83
• Know the natural history and late complications of the muscular dystrophies 2006;27:83
• Formulate a differential diagnosis for a patient who has weakness and an increased serum creatine
kinase activity 2006;27:83 2008:227

2. Pathophysiology
• Know the genetics of dystrophinopathy (Duchenne/Becker muscular dystrophy) 2006;27:83

3. Diagnosis
• Understand the laboratory studies available to diagnose muscle disease of childhood2006;27:83

K. Central nervous system trauma

1. Pathophysiology
• Know that the outcome of a head injury is related to the duration and degree of coma 2007;28:215
• Know the signs and symptoms of spinal trauma 2007:150
• Recognize the clinical features of epidural hematoma 2007;28:215
• Recognize the clinical features of subdural hematoma 2007;28:215

2. Management
• Know the long-term neurologic and behavioral consequences of headtrauma 2007;28:215
• Recognize the neuroendocrine complications of a following head injury 2007;28:215
• Recognize that cerebral edema is a consequence of head trauma 2007;28:215; 305
• Understand the clinical course and management of epidural hematoma 2007;28:215 2008:220:
• Recognize the association of cervical cord injury with head trauma 2007;28:215
• Recognize that intracranial hematomas can occur in the absence of a skull fracture 2007;28:215
• Understand the clinical course and management of subdural hematoma 2007;28:215
• Know the role of pharmacologic therapy in acute spinal cord or craniocerebral trauma 2007:150

L. Neurodiagnostic testing
• Know the value and limitations of neurodiagnostic techniques such as magnetic resonance
imaging, computed tomography, and ultrasonography 2010:38 2010:70

MRI: gold standard imaging test for focal-onset seizures


CT:
US:

Urine toxicology: first unprovoked seizure, but lack of confusion or encephalopathy with a rapid return to a normal
mental status argues against a drug-induced seizure.

LP if partial seizure, fever; if confusion to evaluate for encephalitides

EEG: to look for epilepsy after two unprovoked seizures; In the setting of a single focal seizure, the initial emphasis
should be on determining whether a treatable focal lesion caused the seizure, for which EEG is unlikely to be helpful.

• Understand the value and limitations of neurodiagnostic techniques such as evoked potentials,
electromyography, and electroencephalography 2009:118

For a child who has a first unprovoked seizure safety concerns (no unsupervised time in bathtub or pools, wearing
a helmet while on a bike or scooter) and seizure first aid should be reviewed with the parents and documented in
the chart. Treatment after a single seizure in childhood is not recommended, no matter what EEG shows.
In some cases, retrospective evaluation of children suggests that they have had subtle complex partial seizures or
previously have been found after an unwitnessed seizure. Therefore, it is critical for the physician seeing the child
after an apparent first seizure to obtain a very careful history to be certain that the seizure is, indeed, the first.

The most important diagnostic decision after a first unprovoked seizure is whether to obtain neuroimaging to diagnose
a focal, treatable cause for seizures. Neuroimaging should be obtained after a first unprovoked seizure when the
seizure occurs in the first year after birth, is focal/asymmetric, or begins with a stare and subsequently
generalizes.

Obtaining an EEG is recommended by published practice guidelines. There are two primary problems to keep in mind
when deciding whether to order an EEG:

1) Inter-reader agreement in the interpretation of EEGs is low. The level of training of neurologists reading pediatric
EEGs may be low in some communities, which is a particular problem for neonatal EEGs, and even experienced
EEG readers frequently differ in their visual interpretations.

2) Positive and negative predictive values are not very informative. The sensitivity of EEG varies widely, depending
on who reads it, but at best is approximately 50%; that is, about 50% of children who have epilepsy and 50% of
children who have a first unprovoked seizure that eventually will recur have epileptiform EEGs, but the rest have
normal EEGs. Accordingly, a normal EEG reading does not rule out recurrence or epilepsy and only means that a
recurrent seizure is somewhat less likely. // The specificity after first seizure is, at best, about 70%; that is, about 30%
of children who do not have a seizure recurrence still have epileptiform discharges on the EEG. Thus, a positive
EEG also does not predict epilepsy accurately.

Repeat EEGs with sleep deprivation have been advocated by neurology practice parameters for many years in cases
where the first EEG is read as normal and sleep was not present. The best available evidence suggests, at most,
modest benefit to this practice if used routinely in children. Any benefits of the higher yield should be weighed
against the potential for additional false-positive results, the time for the additional trip to the EEG laboratory,
missed school and work, cost, and the burden of sleep deprivation.

Regardless of EEG results, two or more unprovoked seizures in a child are diagnostic of epilepsy and warrant
treatment and consultation with a neurologist for selection of medication. Because phenobarbital causes sedation and
cognitive and behavioral changes in young children, it would not be used, except in countries where resources for
medical care are very limited. Phenytoin has somewhat erratic absorption and can cause gingival hyperplasia in
children, so it is not first-line therapy for a new diagnosis of epilepsy. Carbamazepine is a good choice for a child
diagnosed with partial epilepsy, but not for generalized epilepsy. Valproic acid is a good choice for both partial and
generalized epilepsy in children older than age 2 years.

The patient’s clinical presentation should guide the diagnostic evaluation. Neuroimaging is essential for any patient
demonstrating focal neurologic findings, changes in behavior or personality, or regression in development. CT of the
brain is useful in detecting structural abnormalities, hemorrhages, calcification, and mass effect. MRI is a better
modality for delineating degenerative disease, gray-white differentiation, inflammation, and infection.

[ For patients manifesting life-threatening signs, the clinician should suspect a neurometabolic
disorder and obtain blood gases and measurements of serum ammonia, lactate, pyruvate, and glucose
to rule out primary metabolic acidosis, hyperammonemia, and hypoglycemia. A CBC, serum chemistry
panel, thyroid panel, and liver transaminase levels are important in evaluating systemic findings such
as growth failure or hepatomegaly. ]

Weakness and hypotonia


• Distinguish among acute and chronic causes of weakness 2007:102
• Know the benefits and limitations of ancillary neurodiagnostic tests in the evaluation of weakness
(eg, serum creatine kinase activity, electromyography 2007:102
• Distinguish between central and peripheral nervous system causes of hypotonia 2008;29:243
2007:118
• Know the differential diagnosis of hypotonia in infants 2008;29:243 2010:150 2007:118
• Know how to evaluate hypotonia in infants 2007:118

Hypotonia: Resistance of muscle to stretch


Phasic tone: Is a rapid contraction in response to a high-intensity stretch
• POSTURAL TONE : Prolonged contraction to a low-intensity stretch

• Hypotonia: Impairment of ability to sustain postural control and movement against gravity

• Weakness: Reduction of maximum power that can be generated by muscle contraction


Hypotonia - Decreased resistance to flexion and extension of extremities

• Central account for 60% to 80% of hypotonia cases


• Hypoxic-Ischemic Injury
• Hypotonic Cerebral Palsy
• Intracranial Hemorrhagic
• Chromosomal Disorders**- Down Synd, Frag X, Prader Willi
• Infants with CENTRAL hypotonia may have:
• Abnormal eye movements/inability to track visually; Fail to imitate facial gestures; Dysmorphic
features
• Fisting/Scissoring on vertical suspension; Hyperactive reflexes
• Early seizures, Apnea or Irregular breathing pattern

• Peripheral causes account for 15% to 30%


• Anterior Horn Cell Disorders
• Spinal Muscular Atrophy
• Traumatic Myelopathy
• Neurogenic Arthogryposis
• Neuromuscular Junction Disorders
• Infantile Botulism; Neonatal acquired myasthenia
• Magnesium Toxicity; Aminoglycoside Toxicity
• Congenital Myopathies
• Nemaline Myopathy; Myotubular Myopathy
• Muscular Dystrophies
• Duchenne’s + Becker’s Muscluar Dystrophy
• Walker-Warburg Disease
• Congenital Myotonic Dystrophy
• Infants with PERIPHERAL Hypotonia may have:
• Alertness to environment, Normal sleep-wake pattern
• Absent reflexes, Fasciculations of tongue, Musc atrophy
• Questions to Explore:
• Family History (ex. Fragile X, DMD)
• Prenatal, Perinatal, Neonatal history
• Maternal exposure to toxins
• Maternal infections (TORCH)
• Mode of Delivery: Breech delivery, Cervical Spinal injury (birth trauma)
• Low APGAR Scores
• Physical Exam:
• Head size, shape
• Dysmorphic features
• Posture, movement
• Often limbs abducted and extended ?
• Plagiocephaly frequently present
• Greek break: “asymmetric distortions of one side of skull” or “Helmet Head”
Horizontal and Vertical suspension tests, Head lag with traction
Hyperlaxity of joints, exaggerated hip abduction

• Older Children: Abnormal stability and movement


– Combat Crawl
– Wide-based gait
– Genu Recurvatum (hyperextended knee)
– Hyperpronation of Feet
– W-Sitting

• Jaw laxity/lip protrusion/eyelid lag


• Chewing dysfunction
• Poor respiratory effort
• Gastro-esophageal reflux
• DTRs :
– Hyperactive (in central conditions)
– Normal/decreased/absent (in peripheral disorders)

Kabuki Synd, , X linked Mental Retardation

Myotonic Dystrophy
• Myotonia (Slow relaxation of muscles after voluntary contraction)

• Autosomal dominant inheritance

• Unstable DNA-trinucleotide repeat on Chromosome 19 --> expand in successive generations

• Usually begin in young adult life


– Weakness of face, distal limb
– Cataracts
– Frontal baldness
Clouded lens

Congenit Myotonic Dystrophy (different)

DMD

• Natural History
– Clinical symptoms around 3-5 years
– Walk late around 18 months
– Walk tip-toe
– Running, hopping, jumping awkward
– Trendelenberg gait (waddling gait)
– Fall easily◊ Gower’s Maneuver to get up
– Lumbar lordosis

• Late Complications
– Wheelchair by 8-12 years
– After losing ambulation, 90% develop scoliosis
– Join contractures
– Deteriorating pulmonary function
– EKG/ECHO changes
– Prognosis: Death by late-teen to mid-20s
• 75% respiratory, 25% left ventricular failure

SMA
• Heterogeneous group of genetic disorders
• Intantile Form: Werdnig-Hoffman Disease

• Disease Mech.: Degeneration of the anterior horn cells in the spinal cord and motor nuclei in brainstem

• Gradual generalized muscle weakness◊ atrophy

• Infantile SMA: Werdnig-Hoffman Disease


– 0 to 6 month old infant with progressively and rapidly expanding weakness
– May have reduction in fetal movement
– Floppiness of limbs and trunk
– Infant in “frog-leg position”
– Affected children never sit or stand unassisted
– Majority will require respiratory support to survive up to age 2

Pompes Disease

Infant botulism
• Mech: Eating foods contaminated by exotoxin of Clostridium Botulinum --> Exotoxin blocks release of
acetylcholine at neuromuscular junction--> cholinergic blockade of skeletal muscle and autonomic nerves
innervating end organs.
• Why are infants and not adults affected?- Ans: Gut pH of infants is not sufficient to kill exotoxin (add further
explanation with pH numbers). Only 20% of cases honey or corn syrup is identified. Affected infant is usually
< 6mo.

• Lives in dusty environment adjacent to construction or agricultural soil disruption

• Prodrome: Constipation, lethargy, poor feeding --> 4-5 days of progressive skeletal muscle weakness--> loss
of DTRs
• Other findings: ptosis, sluggish pupillary light reaction, poor suck, difficulty swallowing, decreased gag reflex
and expressionless face

• Muscle paralysis can lead to respiratory failure!!


• Treatment: Intravenous human botulinum immunoglobulin
• Mech.: Neutralizes circulating botulinum toxin
• Cost: Baby BIG costs $40,000!!!

• Disease Course: Without treatment, usually self-limited and resolves in 2 to 6 weeks with complete recovery.

– CBC
– Rule out sepsis (pan-culture)
– Serum electrolytes
– Liver function tests
– Urine drug screen
– Pearl: “Hypotonic newborn should be considered septic until proven otherwise”
• If hepatomegaly present
– TORCH titers
– Urine culture for cytomegalovirus
– Head ultrasound ◊ look for intracranial hemorrhage or calcifications (CMV) in neonates

• If hypotonia considered central, evaluate for genetic causes


– Send karyotype (especially, if dysmorphic feat. ++)
– Genomic hydribization study for SMA, X-linked diseases, Duchenne’s
– Methylation study for 15q11.2 imprinting (Prader Willi/)

• If complex multisystem involvment◊screen for inborn errors of metabolism


– Plasma amino acid/Urine org. acid (aminoacidopathy, organic acedemia)
– Serum lactate (carbohydrate metabolism, mitochondrial disease)
– Pyruvate
– Ammonia (Urea cycle defects)
– Acylcarnitine profile (Fatty acid oxidation defect)
– Pearl: “Term infant born healthy, but develops floppiness over 12 to 24 hours may have inborn errors
of metabolism”
– Further Metabolic workup:
– VLCFA ◊ Peroxisomal disorders
– Creatine Kinase (CK)◊ Muscular dystrophy
– Carnitine concentration ◊ Carnitine deficiency

• EMG
• Nerve Conduction Study
• Muscle Biopsy
– Immunohistochemical staining
– Electron Microscopy

Peredo, D., Hannibal, M et. al. The Floppy Infant: Evaluation of Hypotonia. Pediatrics in Review. Vol. 30 (2009):
66-76

Fenichel GM. Neonatal Neurology 3rd edition. Churchill Livingston Inc. 1990

Paro-Panjan D, Neubauer D. Congenital hypotonia: is there an algorithm? Journal of Child Neurology; Jun2004,
Vol.19 (6): 439-43

Prasad AN, Prasad C. The floppy infant: contribution of genetic and metabolic disorders. Brain and Development;
Oct 2003, Vol.25(7): 457-76

**

Neurofibromatosis (NF) is an autosomal dominant neurocutaneous disorder of which there are at least two types.
Type 1 has an incidence of approximately 1 in 3,500 people, and its characteristic features are the cutaneous café au
lait macules and benign neurofibromas (Item C181). Type 2 has an incidence of approximately 1 in 40,000, and a
characteristic feature is vestibular schwannoma. Individuals affected by NF-1 are at risk for the development of a
number of complications, including optic pathway tumors, skeletal deformations, speech impediments, and learning
disabilities. In addition, there is variable association with hypertension, short stature, constipation, and chronic
headache. Patients who have NF-2 are not at increased risk for such complications.

The child described in the vignette is at risk for the development of hypertension, particularly during ages 2 through
10 years. Although most patients have idiopathic hypertension, NF-1 can be associated with renal artery stenosis and
pheochromocytoma. This child's health supervision visits should include a detailed history and accurate blood
pressure measurements to ensure early identification of this complication. The child is not at an increased risk for the
development of cataracts, and there is no reason to restrict him from contact sports if he has no other complications.
Because approximately 15% of individuals who have NF-1 develop optic pathway tumors, periodic ophthalmologic
evaluation is important. However, the potential high cumulative radiation dose of annual computed tomography
scanning likely outweighs the benefits; the necessity and frequency of such imaging should be determined in
consultation with the ophthalmologist. Acoustic neuroma, now referred to as vestibular schwannoma, is seen in NF-2,
not NF-1.

American Board of Pediatrics Content Specification:

• Recognize that hypertension is associated with neurofibromatosis

References:
Hersh JH, Committee on Genetics. Health supervision for children with neurofibromatosis. Pediatrics. 2008;121:633-
642.

**

The Condition
Hashimoto encephalopathy (HE) was described first as a condition associated with autoimmune thyroid disease. The
diagnosis is made by demonstrating an abnormal increase in serum antithyroglobulin and antiperoxidase antibodies in
the setting of an encephalopathy. However, the presence of serum antithyroid antibodies does not necessarily correlate
with thyroiditis, and thyroiditis does not necessarily correlate with encephalopathy. Although Hashimoto thyroiditis is
the most common cause of goiter in adolescents, patients who have HE, as in this case, often do not have a goiter.

Recent authors have proposed changing the name of HE to "steroid-responsive encephalopathy associated with
autoimmune thyroiditis" (SREAT). However, even this label is a misnomer because the diagnosis of SREAT does not
require the presence of abnormal thyroid function. Although elevated intrathecal antibodies may represent a more
specific diagnostic criterion, responsiveness to steroids in the presence of serum antithyroid titers is the defining
characteristic at this time. Because HE is a relatively uncommon disorder (estimated prevalence 2 per 100,000), it is
important to exclude more common disorders.

Clinical features of HE include intermittent confusion or change in consciousness, coma, seizures, psychosis,
hallucinations, strokelike episodes, chorea, myoclonus, ataxia, and tremor. Limited neuropathologic studies have
reported lymphocytic infiltration of the parenchyma and leptomeninges, but investigations have not consistently
confirmed the concept that HE is a true vasculitis.

Therapy
Despite any uncertainties regarding pathophysiology, HE is managed commonly as a vasculitis. Current treatment
consists of a pulse of corticosteroid followed by maintenance corticosteroids. However, refractory cases are not
uncommon, and immunosuppressive medications and immune modulation therapies have been implemented in such
patients. Spontaneous remission also has been reported, as in this girl.

Botulism is associated with weakness, cranial nerve palsies, swallowing difficulties, and constipation. Dx: Stool toxin.

Myasthenia gravis usually has a slow, insidious onset with episodic weakness and almost always presents with ptosis
and ophthalmoplegia but intact reflexes and sensation. Muscle diseases rarely present with rapidly progressing
weakness.

** The Underlying Condition


Arterial dissection accounts for 7% to 20% of pediatric AIS and may occur in the anterior (carotid) or posterior
(vertebrobasilar) circulations. Dissection occurs when blood extrudes into the medial layer of the arterial wall through
a tear in the endothelial surface. This abnormal surface allows pathologic thrombus formation, with subsequent
localized arterial occlusion or embolization to downstream arteries. The pathophysiology of the initial vessel injury is
not completely understood. Tethering of the vertebral arteries between the C1 and C2 vertebrae is likely to be relevant
because neck rotation stretches the local segment of the artery. A history of recent trauma often is obtained, but the
trauma usually is trivial and within the normal experiences of childhood. A history of sports injuries or chiropractic
neck manipulation should be sought. Adult evidence suggests that most patients experiencing dissection harbor
ultrastructural connective tissue abnormalities, although few have recognizable clinical syndromes such as Ehlers-
Danlos or Marfan. Specific clinical clues to dissection include the presence of Horner syndrome (carotid dissection) or
neck pain (vertebral dissection).
 If pt is neurologically normal, has no history of prior neurologic illness, and has an unprovoked seizure w/no
evident acute cause, the child has a 24% risk of having another seizure in the next year

 45% risk over the next 14 years, according to one large prospective study

The Specific Condition


X-ALD is an X-linked peroxisomal disorder. Pathologically, it is one of three "dysmyelinating" types of white matter
disease, commonly known as the leukodystrophies. Biochemically, it is a disorder of lipid metabolism resulting in
VLCFA buildup in all tissues. X-ALD causes progressive demyelination and adrenal insufficiency. The gene is
mapped to chromosome region Xp28, and the phenotypic expression is highly variable. In the childhood form,
previously healthy boys of 4 to 10 years of age develop symptoms of ADHD, followed by hearing and visual
problems. This subtype progresses to a vegetative state within 5 years. Adrenomyeloneuropathy features slowly
progressive stiffness and weakness, beginning in the mid-twenties. Corticosteroid replacement therapy may reverse
adrenal insufficiency. Bone marrow transplantation may halt or reverse cerebral demyelination, but is most effective
early in the course and has a 20% risk of morbidity and mortality. Gene therapy may be an option in the future.

Transitory Myasthenic Synd


• Mechanism: Passive placental transfer of antibodies against acetylcholine receptors
• Severity of symptoms correlates with newborn's antibody concentration
• Difficulty feeding and hypotonia are major features
• Occurs within hours of birth or up to 3 days later
• Average duration of symptoms is 18 days and usually resolves
• Diagnosis: Temporary reversal of weakness after edrophonium injection

2007 PREP Neuro:

The developmental regression, acquired microcephaly, and hand-wringing movements


described for the girl in the vignette are typical findings in the X-linked condition Rett syndrome.
Rett syndrome is a neurodegenerative disorder that is associated with developmental
regression as well as generalized tonic-clonic seizures, poor feeding, constipation, sleep
disorder, breath-holding spells, scoliosis, muscle wasting, cardiac arrhythmias, and death in late
adolescence. The diagnosis can be made by testing for the MECP2 gene (chromosome X q28),
a gene for a transcription factor that binds to methylated CpG island and silences transcription.
Deficiency of arylsulfatase A causes the degenerative lysosomal disorder metachromatic
leukodystrophy. Fragile X testing is performed primarily in boys who have cognitive impairment
and dysmorphic features. Hexosaminidase A deficiency causes Tay-Sachs disease.
Accumulation of N-acetyl-aspartic acid in the urine is associated with the leukodystrophy
Canavan disease.

Headache is a frequent primary or secondary complaint in pediatric practice. The clinician must
be able to distinguish primary, benign headaches (ie, migraines or tension headaches) from
secondary, pathologic headaches, which can stem from a tumor, abscess, or other causes of
increased intracranial pressure.
Secondary or ominous headaches are characterized by occipital or frontal pain. The
headache frequently is worse in the morning, after the child has been recumbent at night and
intracranial pressure rises with less effect from gravity. The child may wake in the early morning
with a complaint of head pain. Vomiting may occur at that time, too. Headaches that are
associated with persistent vomiting are worrisome, especially when diarrhea is absent.
Secondary headache also may be precipitated by the Valsalva maneuver.
New-onset neurologic findings in the first 2 to 6 months after onset of pain are especially
concerning for a secondary headache. Specifically, the finding of papilledema, strabismus,
unilateral weakness, or ataxia should alert the practitioner to the likelihood of a structural lesion.
Neurologic deficits such as weakness, strabismus, or ataxia also can be seen rarely with
migraine variants, such as hemiplegic migraine, ophthalmoplegic migraine, or basilar migraine,
respectively, but such atypical headache syndromes are diagnoses of exclusion. Urgent
neuroimaging should be obtained for children who have headache and neurologic deficits.
Computed tomography scan of the head without contrast can be obtained quickly without
sedation to identify a structual lesion causing headache from mass effect. Alernatively, magnetic
resonance imaging of the head can be obtained if the child is stable and monitored closely during
any sedation that may be required.
The headaches exibited by the 17-year-old girl in the vignette, combined with her ataxia, left
hemisensory loss, right dyscoordination, and neck pain, point to a posterior fossa mass. Head
computed tomography scan is the most appropriate next step. Lumbar puncture is
contraindicated when there is suspicion of a mass. Nerve conduction velocities are useful only
to evaluate a peripheral neuropathy, and somatosensory evoked potentials are used to evaluate
demyelination in the peripheral or central nervous system. A toxin would not produce this
constellation of symptoms and signs.
References:
Fisher PG. Help for headaches: a strategy for your busy practice. Contemp Pediatr. 2005;22:34-
41
Haslam RHA. Headaches. In: Behrman RE, Kliegman RM, Jenson HB, eds. Nelson Textbook of
Pediatrics. 17th ed. Philadelphia, Pa: WB Saunders; 2004:2012-2015
Honig PJ, Charney EB. Children with brain tumor headaches. Distinguishing features. Am J Dis
Child. 1982;136:121-124. Abstract available at:
http://www.ncbi.nlm.nih.gov/entrez/query.fcgi?cmd=Retrieve&db=pubmed&dopt=Abstract&list_ui
ds=7064925&query_hl=7&itool=pubmed_DocSum
Lewis DW, Ashwal S, Dahl G, et al, Quality Standards Subcommittee of the American Academy
of Neurology; Practice Committee of the Child Neurology Society. Practice parameter: evaluation
of children and adolescents with recurrent headaches: report of the Quality Standards
Subcommittee of the American Academy of Neurology and the Practice Committee of the Child
Neurology Society. Neurology. 2002;59:490-498. Available at:
http://www.neurology.org/cgi/content/full/59/4/490

The 12-year-old girl described in the vignette has acute recurrent headaches that are consistent
with migraine with aura, ie, classic migraine. Her headaches are short and pounding and have
long pain-free intervals. The dots she sees during episodes are likely scintillating scotomas. The
girl does not display the features of ominous secondary (ie, pathologic) headaches, namely,
papilledema, strabismus, weakness, or ataxia.
Migraines tend to be familial and can be triggered by fatigue, glare, caffeine, fasting, lack of
sleep, menstruation, stress, or other factors. Mood changes can trigger migraine, although
depression tends to produce chronic daily headaches with less migrainous characteristics.
Management of migraines requires both pharmacologic and nonpharmacologic techniques.
After the child and family are reassured about the benign nature of the headache, triggers should
be sought and removed. Daily exercise, routine meals, good hydration, and sufficient sleep are
essential. Other nonpharmacologic measures should be considered, such as relaxation
techniques or biofeedback. These nonpharmacologic measures also are effective in headaches
associated with stress, particularly when chronic. Abortive medications also can be useful to
stop pain, although placebo alone can be associated with cessation of pain in perhaps one third
of children. A recent practice parameter, endorsed by the American Academy of Pediatrics,
reviewed drug treatments for acute headache and found convincing data to support the use of
acetaminophen (15 mg/kg orally), ibuprofen (10 mg/kg orally), or nasal sumatriptan (20 mg if
body mass is 30 kg or more; 5 mg if less than 30 kg). Daily prophylactic medications, such as
amitriptyline, topiramate, cyproheptadine, or valproic acid, are indicated only for the child who
has several disabling migraines every month.
Evidence to support the use of subcutaneous sumatriptan or oral zolmitriptan in children is
limited at this time. Intramuscular meperidine and intranasal butorphanol are both narcotics that
have addictive potential and are not indicated for treatment of childhood migraine.

Cerebral palsy is an umbrella term covering a group of nonprogressive, but often changing,
motor impairment syndromes due to lesions or anomalies of the brain that arise in the early
stages of its development. Some 2 to 3 per 1,000 children have cerebral palsy in the United
States. Although it may be difficult to determine the exact cause of the motor impairment for an
individual child, several clinical situations are associated with increased risk of cerebral palsy,
including brain malformation, chromosomal abnormalities, intrauterine growth restriction,
prematurity, birth hypoxia, and postnatal events such as traumatic injury and meningitis.
The motor manifestations of cerebral palsy are treated best by an interdisciplinary team of
clinicians who can address medical and surgical management of tone and joint contractures as
well as therapists who can address therapeutic issues such as range of motion and equipment
needs.
As described for the boy in the vignette, children who have cerebral palsy frequently have
other coexisting conditions, such as cognitive disability or mental retardation, skeletal
malformations, visual impairment, communication disorders, seizures, behavioral concerns,
abnormal oral-motor function, and poor growth and nutrition. Screening for and treatment of
associated conditions can support his motor, social-adaptive, and cognitive development. He
most likely will have difficulty with feeding and constipation, and it is helpful to provide anticipatory
guidance for the family to ensure that he receives adequate nutrition.
Cerebral palsy is a nonprogressive condition; it is not characterized by developmental
regression. Cerebral palsy also is not associated with a higher risk of skin or cardiac conditions.
The boy may require tendon releases in the future, but no contractures are noted on this
physical examination, and he is very young to develop this complication of cerebral palsy.
References:
Cooley WC; American Academy of Pediatrics Committee on Children With Disabilities. Providing
a primary care medical home for children and youth with cerebral palsy. Pediatrics.
2004;114:1106-1113. Available at:
http://pediatrics.aappublications.org/cgi/content/full/114/4/1106
Green L, Greenberg GM, Hurwitz E. Primary care of children with cerebral palsy. Clin Family
Pract. 2003;5:467-491
Johnston MV. Cerebral palsy. In: Behrman RE, Kliegman RM, Jenson HB, eds. Nelson
Textbook of Pediatrics. 17th ed. Philadelphia, Pa: WB Saunders Co; 2004:2024-2025
Copyright

Evaluation of Ataxia

Elaine A. Dinolfo, MD*


*
The Children’s Hospital at Montefiore Bronx, NY

Ataxia- impaired coordination of movement and balance


associated with dysfunction of the cerebellum or the sensory or motor pathways connecting to it.

Thorough H&P: The history must include the onset and duration of the symptom; family history of migraine or
neurologic disease; recent infection, seizure, or head injury; and possible environmental exposure to heavy metals,
gases, solvents, or medications. It is important to ask about other signs and symptoms, such as headache, vomiting,
photophobia, vertigo, and altered mental status.

Distinguishing whether a young child has age-appropriate clumsiness or is truly ataxic requires familiarity with
developmental milestones. The child should be examined in an upright position with attention to mental status,
balance, presence of nystagmus, deep tendon reflexes, and muscle tone and strength.

Ddx: Muscle weakness from a myopathy or neuropathy may present with an altered gait and unstable balance, but it
should be recognized on the basis of diminished muscle strength and deep tendon reflexes. Conversely, children who
have spasticity may exhibit an unsteady, narrow-based gait, but they will demonstrate increased muscle tone and
brisk reflexes. Movement disorders such as chorea and myoclonus, unlike ataxia, are evident even when the child is at
rest.

The examiner must look for evidence of increased intracranial pressure or infection by checking for abnormal vital
signs, meningismus, and papilledema. With a patient is old enough to cooperate, the neurologic examination should
evaluate cranial nerve function, heel-to-knee movement, finger-to-nose movement, tandem gait, rapid alternating
movement, and speech. It is particularly important to listen for dysarthria, specifically the scanning speech pattern
associated with cerebellar dysfunction.

Dysfunction of the cerebellar vermis presents with truncal ataxia—imbalance in the sitting position and head
titubation. Dysfunction of the cerebellar hemispheres presents with a gait that veers toward the involved side and
dysmetria of the ipsilateral extremity. Attenuation of sensory input because of peripheral nerve or posterior column
damage leads to a careful high-stepping gait and a positive Romberg sign. Children who have sensory ataxia have
problems with fine finger movements rather than dysmetria.

A typical presentation of acute ataxia is the sudden onset of a wide-based gait, with swerving and unsteadiness that is
recognized easily by parent and physician. The most common causes are drug ingestion and postinfectious
cerebellitis. The differential diagnosis includes head trauma, vascular events, brain tumor, hydrocephalus, the Miller
Fisher variant of Guillain-Barré syndrome (ataxia, ophthalmoplegia, and areflexia), labyrinthitis, seizure, and
conversion reaction. Drugs associated with ataxia include phenytoin, carbamazepine, sedatives, hypnotics, and
phencyclidine. Intoxication with alcohol, ethylene glycol, hydrocarbon fumes, lead, mercury, or thallium also may
present with ataxia. A variety of pathogens can cause postinfectious cerebellitis, but it is observed most frequently
after varicella.

The evaluation of the child who has acute ataxia is based on the history and findings on physical examination, but it
also should include a complete blood count, measurement of electrolytes, toxicologic screening of blood and urine,
brain imaging, and lumbar puncture. If magnetic resonance imaging or computed tomography demonstrates a mass
lesion, hydrocephalus, or other intracranial abnormality, cerebrospinal fluid (CSF) evaluation may be deferred. The
presence of cells in the CSF may indicate infection, and elevated CSF protein levels are seen with Guillain-Barré
syndrome and multiple sclerosis.

If the initial evaluation fails to establish a diagnosis, what appears to be acute ataxia actually may be the initial
presentation of a more chronic process. Basilar migraine, seizure, and some rare metabolic disorders (Hartnup
disease, maple syrup urine disease, pyruvate decarboxylase deficiency) may manifest with acute intermittent episodes
of ataxia. Evaluation again is based on the history, but it may include brain imaging, electroencephalography, and
electromyography. If metabolic disease is suspected, evaluation of amino acids, acid/base balance, lactate, pyruvate,
ammonia, and ketones may be helpful.

The slow evolution of ataxia in a previously healthy child warrants rapid evaluation. The most common causes of
progressive ataxia are brain tumors (medulloblastoma, cerebellar astrocytoma, brain stem glioma, ependymoma) and
degenerative spinocerebellar diseases, some of which present in infancy and others of which are delayed in onset until
adulthood.

Friedreich ataxia, an autosomal recessive disorder linked to a defect on chromosome 9, presents in the first decade of
life with ataxia, pes cavus, scoliosis, and areflexia.

Ataxia telangiectasia, also autosomal recessive, is marked by an abnormality on the long arm of chromosome 11 and is
characterized by progressive truncal ataxia, oculocutaneous telangiectasias, and recurrent infections of the sinuses and
lungs. // If magnetic resonance imaging of a child who has progressive ataxia fails to disclose an intracranial mass,
referral for a detailed metabolic and genetic evaluation is warranted.

References:
Dinolfo E. Evaluation of ataxia. Pediatr Rev. 2001;22:177-178. Available at:
http://pedsinreview.aappublications.org/cgi/content/full/22/5/177
Johnston MV. Movement disorders: ataxias. In: Behrman RE, Kliegman RM, Jenson HB, eds.
Nelson Textbook of Pediatrics. 17th ed. Philadelphia, Pa: WB Saunders Co; 2004:2019-2023
MacDonald GP. Ataxia of childhood. In: Berg BO, ed. Child Neurology: A Clinical Manual. 2nd ed.
Philadelphia, Pa: JB Lippincott Co; 1994:287-305
Copyright

Critique: 70 Preferred Response: B


The sudden and self-limited ataxia with nystagmus, but without vomiting or loss of
consciousness, followed by some residual horizontal nystagmus described for the girl in the
vignette is typical of benign paroxysmal vertigo. Benign paroxysmal vertigo is characterized by
abrupt, brief episodes of vertigo with ataxia in children ages 1 to 3 years. The child may appear
frightened, have pallor, and may indicate feeling dizzy. Rapid eye movements or nystagmus
may be observed if the eyes are open. Benign paroxysmal vertigo is believed to be a migraine
variant.
Vertigo is defined as an illusion of movement, most often a sensation of rotation. The
simplest approach to evaluating this complaint is to consider its temporal pattern and whether
hearing is affected. Acute vertigo without hearing loss may stem from benign paroxysmal vertigo
in toddlers, vestibular neuronitis in adolescents, labyrinthine concussion, migraine, seizures
(rarely), and benign paroxysmal positional vertigo (BPPV). Most of these disorders last minutes
to sometimes hours, although BPPV lasts only 5 to 20 seconds and occurs with changes in
head position. BPPV can be elicited by the Hallpike-Dix maneuver. The child is moved abruptly
from a sitting to lying posture, with the head hanging 45 degrees below the horizontal and rotated
45 degrees to one side, evoking vertigo and nystagmus.
Acute vertigo lasting hours to days accompanied by hearing loss may be due to labyrinthitis,
otitis media, perilymphatic fistula, temporal bone fracture, and Ménière disease. Ménière disease
is a recurrent disorder that involves tinnitus and progressive hearing loss.
Continuous vertigo over days to weeks may indicate a cholesteatoma; posterior fossa
tumor; vestibular schwannoma (associated with neurofibromatosis II); demyelinating disease; or
drug toxicity from aminoglycosides, isoniazid, furosemide, or phenytoin.
The 2-year-old child in the vignette has not displayed headache or cranial neuropathies,
making basilar migraine unlikely. The spontaneous and rapid resolution of her symptoms
excludes cerebellar hemorrhage and phenytoin intoxication. No loss of consciousness has been
observed, making seizure unlikely.
References:
MacGregor DL. Vertigo. Pediatr Rev. 2002;23:10-16. Available at:
http://pedsinreview.aappublications.org/cgi/content/full/23/1/10

Sensorineural hearing loss is the most common long-term sequela of bacterial meningitis. It has
been described in up to 30% of patients who have pneumococcal disease and 10% of those
who have meningococcal meningitis. The hearing loss is due to either a labyrinthitis or direct
inflammation of the auditory nerve. Patients who have significant hearing loss often demonstrate
significant truncal ataxia during their recovery because of middle ear inflammation. Other longterm
sequelae include developmental or learning delays, behavioral problems, seizures,
hydrocephalus, and cerebral palsy.
To decrease the risk of long-term neurologic sequelae, the American Academy of Pediatrics
recommends the administration of adjunctive therapy with dexamethasone for the treatment of
bacterial meningitis due to Haemophilus influenzae type B and consideration of treatment for
pneumococcal meningitis in infants and children older than 6 weeks of age. If dexamethasone is
used, it should be given regardless of the clinical severity of the illness and administered as
soon as possible in the course of treatment; best results occur when the first dose is
administered prior to any antimicrobial therapy.
Other acute complications of bacterial meningitis include the syndrome of inappropriate
antidiuretic hormone secretion, seizures, and increased intracranial pressure. Fever may be
present for 5 to 7 days during the illness or may disappear if dexamethasone therapy is used,
only to return after the corticosteroids are stopped. Less common but more serious acute
complications include infarction of the brain or spinal cord, cranial nerve palsy, cerebral
herniation, thrombosis of the sagittal sinus, subdural effusions, and brain abscess formation.
References:
Prober CG. Central nervous system infections. In: Behrman RE, Kliegman RM, Jenson HB, eds.
Nelson Textbook of Pediatrics. 17th ed. Philadelphia, Pa: WB Saunders Co; 2004:2038-2047
Koomen I, Grobbee DE, Roord JJ, Donders R, Jennekens-Schinkel A, Van Furth AM. Hearing
loss at school age in survivors of bacterial meningitis: assessment, incidence, and prediction.
Pediatrics. 2003;112:1049-1053. Available at:
http://pediatrics.aappublications.org/cgi/content/full/112/5/1049
Feigin RD, Watson JT, Gerber SI. Use of corticosteroids in bacterial meningitis. Pediatr Infect
Dis J. 2004;23:355-357
Copyright
Increased intracranial pressure (ICP) is an emergency that must be recognized quickly by
pediatricians. Elevated ICP often is insidious and nonspecific initially. Among affected school-age
children, declining academic performance, fatigue, behavioral changes, and vague intermittent
headaches are common. Over time, morning headaches, especially pain at the occipital or
frontal region, along with vomiting and lethargy ensue. Horizontal diplopia becomes apparent
once the abducens nerve becomes compressed by the tentorium. Papilledema may develop if
the pressure is longstanding, but the optic fundus can appear normal early in the course of
increased ICP. Hypertension with bradycardia and variable respiratory pattern (Cushing triad)
may indicate critically elevated ICP that is nearing cerebral herniation.
Irritability, anorexia, failure to thrive, and even developmental regression can be early signs
of increased ICP in infants. Increased ICP may lead to macrocephaly, splitting of the cranial
sutures, or a bulging anterior fontanelle. The “setting sun” sign, a downward deviation of the
eyes, may be seen with attendant hydrocephalus.
Once there is concern about elevated ICP, immediate neuroimaging is required with
computed tomography (CT) scan of the head or magnetic resonance imaging (MRI) of the brain.
CT scan without contrast can be performed in just minutes, without conscious sedation.
Although MRI provides better definition of the underlying pathology, such neuroimaging takes
about 30 or more minutes and may require sedation. Either modality can determine whether
there is increased ICP from a mass lesion.
The hypertension, bradycardia, and right abducens paresis reported for the child in the
vignette are worrisome for increased ICP, and she should receive a CT scan of the brain for
evaluation. Cerebral angiography only delineates the intracranial vasculature. Positron emission
tomography and single-photon CT scan determine whether a lesion is metabolically active (eg,
tumor or infection). Single-photon CT scan may be useful in identifying the structural origin of
epilepsy immediately following a seizure. Electroencephalography is used to evaluate seizures
and level of consciousness, but has limited, if any, utility in the delineation of mass lesions of the
brain.
References:
DiCarlo JV, Frankel LR. Stabilization of the critically ill child: neurologic stabilization. In: Behrman
RE, Kliegman RM, Jenson HB, eds. Nelson Textbook of Pediatrics. 17th ed. Philadelphia, Pa:
WB Saunders Co; 2004:308-311
Larsen GY, Goldstein B. Consultation with the specialist: increased intracranial pressure. Pediatr
Rev. 1999;20:234-239. Available at:
http://pedsinreview.aappublications.org/cgi/content/full/20/7/234
Copyright

Critique: 102 Preferred Response: C


In general, muscular weakness tends to be greater
proximally than distally, affecting most the hips and shoulders. Creatine kinase is often elevated
in myopathy. Neuropathy also produces symmetric weakness, but the deficit is greater distally
than proximally and associated with hyporeflexia and sometimes, sensory loss. Nerve
conduction velocities in neuropathy demonstrate diminished velocity or amplitude.
Neuromuscular junction problems tend to worsen throughout the day, and in children are more
notable rostrally than caudally. Myelopathy produces weakness inferior to a spinal lesion, often
with a sensory deficit inferior to a specific dermatome. Weakness associated with brainstem
processes often is asymmetric and characterized by cranial neuropathies. Cerebral processes
with weakness produce hemiparesis and unilateral hyperreflexia and may cause visual field loss
or speech problems.
Acute causes of weakness include stroke (cerebrum or brainstem); acute intermittent
porphyria, diphtheria, poliomyelitis, West Nile virus infection, or tick paralysis (neuropathy);
organophosphate poisoning or botulism (neuromuscular junction); and periodic paralysis and
acute infectious myositis (muscle). Subacute processes include tumor or abscess (cerebrum or
spine), transverse myelitis (spine), myasthenia gravis (neuromuscular junction), Guillain-Barré
syndrome (nerve), and polymyositis and dermatomyositis (muscle). Muscular dystrophies tend
to present chronically.
The 7-year-old girl described in the vignette displays weakness that is proximal more than
distal, as well as elevated creatine kinase concentrations, suggesting a myopathy. The 4-week
duration plus the erythematous malar rash raise suspicion about dermatomyositis. Muscle
biopsy can demonstrate the fiber type variability, central nuclei that are vesicular, infiltration with
inflammatory cells, and necrosis that characterize the condition.
The erythrocyte sedimentation rate is a nonspecific test; it is elevated in myopathy,
infections, and multiple other disorders. This child does not have the chronic course of the three
most common dystrophies (Duchenne, Becker, and fascioscapulohumeral). Therefore,
polymerase chain reaction testing for dystrophin or genetic testing for fascioscapulohumeral
dystrophy is not warranted. Serum antinuclear antibody may be positive in patients who have
dermatomyositis, but it is a nonspecific finding. It is most useful in the evaluation of patients who
may have systemic lupus erythematosus, but lupus will not produce the myopathic weakness
reported for this girl.
References:
Jacobson RD. Approach to the child with weakness or clumsiness. Pediatr Clin North Am.
1998;45:145-168. Abstract available at:
http://www.ncbi.nlm.nih.gov/entrez/query.fcgi?cmd=Retrieve&db=pubmed&dopt=Abstract&list_ui
ds=9491091&query_hl=14&itool=pubmed_docsum
Roland EH. Muscular dystrophy. Pediatr Rev. 2000;21:233-238. Available at:
http://pedsinreview.aappublications.org/cgi/content/full/21/7/233
Sarnat HB. Neuromuscular disorders. In: Behrman RE, Kliegman RM, Jenson HB, eds. Nelson
Textbook of Pediatrics. 17th ed. Philadelphia, Pa: WB Saunders Co; 2004:2053-2082
Turner TL, Boom JA. Index of suspicion. Case 1. Pediatr Rev. 2000;21:389-392. Available at:
http://pedsinreview.aappublications.org/cgi/content/full/21/11/389
Copyright

The limpness or floppiness reported for the infant in the vignette points to hypotonia, which often
is disproportionate to weakness. The trunk and extremities of the infant who has hypotonia hang
limply when the child is suspended horizontally. When the clinician suspends the child vertically
by holding the infant’s axillae, the baby will “slip through” the examiner’s hands, rather than
adducting his or her shoulders.
Hypotonia is found in a wide variety of pathophysiologic processes arising throughout the
motor system. Cerebral, or central, causes of hypotonia include chromosomal abnormalities
(Down syndrome, Turner syndrome, Prader-Willi syndrome), metabolic abnormalities
(hypothyroidism, leukodystrophies), sepsis, and perinatal trauma (hypoxia-ischemia or
intracranial hemorrhage). The most worrisome cause of hypotonia localizing to the brainstem
and spine is spinal muscular atrophy. Neuropathy is rarely a cause of infant hypotonia
(congenital polyneuropathy, familial dysautonomia), but peripheral hypotonia commonly stems
from the neuromuscular junction (botulism, myasthenia gravis, or myasthenic syndrome) or
muscle (muscular dystrophy and congenital myopathies).
Clues on physical examination often help lead to the diagnosis. Depressed or absent deep
tendon reflexes are seen with spinal muscular atrophy, neuropathies, and even botulism.
Tongue fasciculations are common with spinal muscular atrophy. Botulism produces rostral
greater than caudal weakness. Low birthweight, almond-shaped palpebral fissures, and
hypogonadism are typical of Prader-Willi syndrome.
Laboratory investigations are targeted toward the disorders suggested by physical
examination findings. The child in the vignette has physical findings typical for Prader-Willi
syndrome; fluorescent in situ hybridization testing for the gene locus at the 15q11-13
chromosomal region is indicated. Arylsulfatase A testing assesses for metachromatic
leukodystrophy, which develops later in the first postnatal year. Measuring thyroid-stimulating
hormone is not likely to be useful because the boy lacks signs of hypothyroidism.
Electromyography and nerve conduction studies are used to assess a congenital myopathy or
spinal muscular atrophy. Magnetic resonance imaging of the brain would be useful if there were
a history of perinatal trauma or concern about cerebral dysgenesis.
References:
Berg BO. The hypotonic infant. In: Child Neurology: A Clinical Manual. 2nd ed. Philadelphia, Pa:
JB Lippincott Co; 1994:279-286
Stiefel L. In brief: hypotonia in infants. Pediatr Rev. 1996;17:104-105
Copyright

Occult spinal dysraphism (spina bifida occulta) can be insidious and nonspecific upon initial
presentation. The pathology can include syringomyelia, diastematomyelia, a spinal cord tethered
to the inferior thecal sac, sacral agenesis, or dermoid sinus. Dermoid sinuses pass from the
skin through the dura, acting as conduits for the entry of bacteria into the nervous system.
Clinical manifestations of occult spinal dysraphism can be subtle. In some instances, a
lipoma, patch of hair, hemangioma, or discoloration of skin lies over the lumbosacral spine. A
sacral dimple superior to the gluteal cleft or lateral to the cleft with displacement of the gluteal
fold, as described for the boy in the vignette, should raise suspicion for dysraphism. Children
later may develop high-arched feet, discrepancy in strength between lower extremities, or a gait
abnormality. Back pain or absent perineal sensation, urinary incontinence, lower urinary tract
infection, or fecal soiling also may occur.
Children younger than age 6 months, whose vertebral bones have not ossified fully, can be
evaluated for spina bifida by ultrasonography. Older children require initial evaluation with
magnetic resonance imaging of the spine. Radiographs might be helpful to evaluate
accompanying constipation, but will not evaluate dysraphism adequately. Urinary tract
abnormalities should be managed after the dysraphism is identified, and the management
approach is similar to that in other children who have myelodysplasia. Ultrasonography of the
kidneys, ureter, and bladder might be helpful, as can urine culture or voiding
cystourethrography, in evaluation of the urinary tract.

The possibility of occult or overt spinal trauma should be considered with any injury. For any
notable body trauma, the cervical and lower spine should be immobilized until spinal trauma has
been eliminated as a possibility. Common sites for fracture dislocation spinal cord injuries are C1
through C2, C5 through C6, and T12 through L1 spinal segments. Fracture dislocations of the
vertebral column are the most common causes of spinal cord injury, although injury can be as
severe as complete cord transection without any visible trauma on radiography. Sudden
whiplash (flexion-extension) injuries may contuse (Item C150A) or lacerate the cervical cord.
Typical symptoms and signs of spinal trauma include focal pain along the spinal axis or any
weakness or sensory loss in the extremities or trunk. “Spinal shock” is characterized by loss of
all voluntary movement and sensation inferior to the site of injury. Reflex function in the cord is
lost temporarily, resulting in ileus and bladder distention.
While the spine is immobilized, the child should undergo a thorough neurologic examination.
If there is high index of suspicion for spinal trauma, as for the girl described in the vignette,
prompt initiation of methylprednisolone 30 mg/kg administered intravenously over 1 hour,
followed by 5.4 mg/kg per hour the next 23 hours, is a protocol that can lead to improved
neurologic outcomes. The cervical spine typically is evaluated by radiographs (including lateral,
anterior-posterior, and odontoid views), but if there is lingering concern, computed tomography
scan or magnetic resonance imaging is warranted.
Intravenous dexamethasone is useful for spinal cord compression with tumors, but its
efficacy has not been investigated fully for spinal trauma. Anticonvulsants sometimes are
administered prophylactically after severe head trauma, but this is controversial. Mannitol is
useful only if the patient displays signs of increased intracranial pressure. Low-molecular weight
heparin to prevent deep venous thrombosis might be useful if the patient is not expected to
regain ambulatory status for several days.
References:
Bracken MB, Shepard MJ, Collins WF, et al. A randomized, controlled trial of methylprednisolone
or naloxone in the treatment of acute spinal-cord injury. Results of the Second National Acute
Spinal Cord Injury Study. N Engl J Med. 1990;322:1405-1411. Abstract available at:
http://www.ncbi.nlm.nih.gov/entrez/query.fcgi?cmd=Retrieve&db=pubmed&dopt=Abstract&list_ui
ds=2278545&query_hl=19&itool=pubmed_docsum
Haslam RHA. Spinal cord disorders: spinal cord trauma. In: Behrman RE, Kliegman RM, Jenson
HB, eds. Nelson Textbook of Pediatrics. 17th ed. Philadelphia, Pa: WB Saunders Co; 2004:2050
Temkin NR, Dikmen SS, Wilensky AJ, Keihm J, Chabal S, Winn HR. A randomized, double-blind
study of phenytoin for the prevention of post-traumatic seizures. N Engl J Med. 1990;323:497-
502. Abstract available at:
http://www.ncbi.nlm.nih.gov/entrez/query.fcgi?cmd=Retrieve&db=pubmed&dopt=Abstract&list_ui
ds=2115976&query_hl=21&itool=pubmed_docsum
Copyright

Organisms from the genus Citrobacter cause a chronic form of bacterial meningitis in the
neonate that leads to brain abscess formation in approximately 80% of infected infants. The
abscesses tend to be large multiloculated areas that can be difficult to treat. The exact
mechanism for the invasiveness of Citrobacter and its propensity to cause brain abscesses is
unknown.
Citrobacter are gram-negative enteric rods closely related to Salmonella that include at least
11 species, the most common of which are C freundii and C koseri. The organisms can be
passed vertically from mother to infant at delivery, but colonization from nosocomial sources
(eg, hand contamination, contaminated infant formula) has been described. Most organisms
from the genus Citrobacter are resistant to ampicillin but susceptible to the later-generation
cephalosporins and aminoglycosides. Treatment of brain abscesses (Item C172A) usually
involves long-term antimicrobial therapy (4 to 6 weeks after sterilization of the cerebrospinal
fluid) with a combined regimen that includes a third- or fourth-generation cephalosporin and an
aminoglycoside. The abscess area frequently must be drained, and follow-up cranial imaging is
used to help assess the duration of therapy. Patients who have multiloculated abscess usually
have significant developmental problems.
Although Escherichia coli, Klebsiella pneumoniae, Listeria monocytogenes, or Streptococcus
agalactiae (group B Streptococcus) can cause a brain abscess, Citrobacter is most likely to be
responsible for the indolent course described for the neonate in the vignette that includes brain
abscess formation. Similar to Citrobacter, Enterobacter sakazakii causes neonatal meningitis
with brain abscess formation, but it is encountered less frequently than Citrobacter.
References:
Boyce TG, Gruber WC, Fisher RG. Citrobacter. In: Feigin RD, Cherry JD, Demmler GJ, Kaplan
SL, eds. Textbook of Pediatric Infectious Diseases. 5th ed. Philadelphia, Pa: WB Saunders Co;
2004:1423-1426
Goodkin HP, Harper MB, Pomeroy SL. Intracerebral abscess in children: historical trends at
Children’s Hospital Boston. Pediatrics. 2004;113:1765-1770. Available at:
http://pediatrics.aappublications.org/cgi/content/full/113/6/1765

Metabolic disorders, degenerative diseases, and infections can produce movement disorders,
but drug exposures also should be considered as causes of ataxia, tremor, and dystonia. Ataxia
can be triggered by anticonvulsant toxicity from phenytoin, phenobarbital, and even
carbamazepine. Alcohol and thallium also can lead to ataxia. Drugs that can cause tremor
include amphetamines, valproic acid, neuroleptics such as phenothiazines, tricyclic
antidepressants, caffeine, and theophylline. Stimulant medications can unmask Tourette
syndrome, but by themselves do not produce tics.
Certain drugs are capable of producing an acute dystonic reaction in children. Therapeutic
doses of phenytoin or carbamazepine rarely cause progressive dystonia in children who have
epilepsy and an underlying structural abnormality of the brain. Children may have an
idiosyncratic reaction to neuroleptic drugs, characterized by acute dystonic posturing that may
be confused with encephalopathy. Intravenous diphenhydramine, 1 to 2 mg/kg per dose, may
reverse the drug-related dystonia rapidly. Severe rigidity combined with high fever and delirium
also may occur as part of the neuroleptic malignant syndrome a few days after the initiation of
neuroleptic drugs.
The 9-year-old girl described in the vignette is experiencing a classic acute dystonic
reaction, with posturing and even oculogyric crisis. Metoclopramide is in the family of neuroleptic
drugs, although used most often for nausea or gastrointestinal motility and most likely is the
cause of the dystonic reaction. The antiemetics lorazepam and ondansetron do not induce
movement disorders, and diphenhydramine is used to reverse dystonia from metoclopramide.
Aprepitant is a new substance P/neurokinin-1 antagonist used for chemotherapy-induced
nausea and vomiting. It can cause weakness or dizziness, but does not cause movement
disorders.

S-ar putea să vă placă și